Вы находитесь на странице: 1из 94

1

Case 3: 65F with insomnia

Common Causes of Insomnia in the Elderly


Issues that may lead to an environment that is not conducive to sleep.
• Specific examples include: noise or uncomfortable bedding.
• You can teach the patient sleep hygiene techniques that will increase the likelihood
of a restful night's sleep.

Question the use of prescription, over-the-counter, alternative, and recreational drugs


that might be affecting sleep.
• Patients should be counseled to avoid caffeine and alcohol for four to six hours
before bedtime.

Sleep apnea is common in the elderly, occurring in 20-70% of elderly patients.


• Obstruction of breathing results in frequent arousal that the patient is typically not
aware of; however, a bed partner or family member may report loud snoring or
cessation of breathing during sleep.

In restless leg syndrome, the patient experiences an irresistible urge to move the legs,
often accompanied by uncomfortable sensations.
In periodic leg movement and REM sleep behavior disorder, the patient experiences
involuntary leg movements while falling asleep and during sleep respectively.
• As in sleep apnea, the sleeper is often unaware of these behaviors and a bed partner
or family member may need to be asked about these movements.

Disturbances in the sleep-wake cycle include jet lag and shift work.
Patients with depression and anxiety commonly present with insomnia.
• Any patient presenting with insomnia should be screened for these disorders.

Patients with shortness of breath due to cardiorespiratory disorders often report that
these symptoms keep them awake.

Pain or pruritus may keep patients awake at night.


Those with GERD may report heartburn, throat pain, or breathing problems.
• These patients may also have trouble identifying what awakens them.
• Detailed questioning may be needed to elicit the symptoms of this disorder.

Elderly patients with hyperthyroidism frequently do not present with typical


symptoms such as tachycardia or weight loss and laboratory studies may be required to
detect this problem.
2

Circadian rhythms change, with older adults tending to get sleepy earlier in night.
In advanced sleep phase syndrome (ASPS), this has progressed to the point where the
patient becomes drowsy at 6 to 7 PM. If they go to sleep at this hour, they sleep a
normal 7-8 hours, waking at 3 or 4 am. However, if they try to stay up later, their
advanced sleep/wake rhythm still causes them to awaken at 3 or 4 am. This can be
difficult to distinguish from insomnia.

Common causes of insomnia in the elderly:


1. Environmental problems
2. Drugs/alcohol/caffeine
3. Sleep apnea
4. Parasomnias: restless leg syndrome/periodic leg movements/REM sleep behavior
disorder
5. Disturbances in the sleep-wake cycle
6. Psychiatric disorders, primarily depression and anxiety
7. Symptomatic cardiorespiratory disease (asthma/chronic obstructive pulmonary
disease/congestive heart failure)
8. Pain or pruritus
9. Gastroesophageal reflux disease (GERD)
10. Hyperthyroidism
11. Advanced sleep phase syndrome (ASPS)

Sleep hygiene:
Habits: Fix a bedtime and awakening time. Avoid napping during the day or limit to 30-
45 minutes. Avoid alcohol 4-6 hrs before bedtime. Avoid caffeine 4-6 hrs before
bedtime. Avoid heavy, spicy, or sugary foods 4-6 hours before bedtime. Exercise
regularly, but not right before bed.

Environment: Use comfortable bedding, find comfortable temperature, keep room


ventilated. Block out all distracting noise, eliminate light, reserve the bed for sleep and
sex.

Getting ready: Light snack before bed (warm milk and foods high in tryptophan) can
help. Practice relaxation techniques. Establish pre-sleep ritual (bath, reading, etc.), get
into favorite sleeping position.

Most people wake up one or two times a night for various reasons. If you find that you
get up in the middle of night and cannot get back to sleep within 15-20 minutes, then do
not remain in the bed "trying hard" to sleep. Get out of bed. Leave the bedroom. Read,
3

have a light snack, do some quiet activity, or take a bath. You will generally find that
you can get back to sleep 20 minutes or so later. Do not perform challenging or
engaging activity such as office work, housework, etc. Do not watch television.

A Word About Television


Many people fall asleep with the television on in their room. Watching television before
bedtime is often a bad idea. Television is a very engaging medium that tends to keep
people up. We generally recommend that the television not be in the bedroom. At
the appropriate bedtime, the TV should be turned off and the patient should go to
bed. Some people find that the radio helps them go to sleep. Since radio is a less
engaging medium than TV, this is probably a good idea.

Treatments for Primary Insomnia in the Elderly


Of the behavioral treatments, many of which may be of some assistance in the elderly,
only sleep restriction/sleep compression therapy and multi-component cognitive-
behavioral therapy have met evidence-based criteria for efficacy.

Cognitive Behavioral Therapy for Insomnia (CBT-I)
CBT-I (which includes sleep


hygiene instruction, stimulus control, and sleep restriction with cognitive restructuring)
has been shown to be most effective. CBT-I combines different behavioral treatments,
resulting in improvements lasting up to two years. Examples include:
• Sleep restriction therapy: The patient is told to reduce his or her sleep/in-bed time to
the average number of hours the patient has actually been able to sleep over the
last 2 weeks (as opposed to the number of hours spent in bed [awake plus
asleep]). As sleep efficiency increases, time allowed in bed is
increased gradually by 15- to 20-minute increments approximately once every 5
days (if improvement is sustained) until the individual's optimal sleep time is
obtained.
• Sleep compression therapy: The patient is counseled to decrease the amount of time
spent in bed gradually to match total sleep time rather than making an
immediate substantial change.

Pharmacological Therapy
All drugs for the treatment of insomnia can be associated with side effects--particularly
prolonged sedation and dizziness--that can result in the risk of injuries and confusion.
Non-benzodiazepines (e.g., zolpidem [Ambien]) and melatonin-receptor agonists
are the safest and most efficacious hypnotic drugs currently available.
Benzodiazepines can be effective but have more complications and the additional risk
of addiction.
Antihistamines, antidepressants, anticonvulsants, and antipsychotics are associated
4

with more risks than benefits in older adults.

Combining CBT-I and pharmacological therapy can be helpful in some patients.

The evidence base for exercise as a treatment for insomnia is less extensive. Despite
this, there are many other reasons to encourage regular physical activity in the elderly,
assuming there are no other contraindications to such activity.

Medical Conditions Associated with Depression


A number of diseases either cause depressive symptoms or have depression as a
comorbidity at higher rates than would be normally expected.
In looking for the causes and associations of depression, first consider the common
conditions. Then think about the very serious diseases that you don't want to miss.
Beyond that, there's a very wide range of diagnoses that can look like depression:
Hypothyroidism: About five percent of the U.S. population has
hypothyroidism. Checking the level of thyroid stimulating hormone (TSH) would help
make the diagnosis. Hypothyroidism can be treated with thyroid-
replacement medications such as triiodothyronine (T3) and/or levothyroxine (T4). Once
TSH levels are returned to the normal range, the symptoms of depression often
subside.
Parkinson's disease: Up to 60 percent of people with this disorder experience mild or
moderate depressive symptoms. Although several reports have shown a link between
depressive symptoms and Parkinson's disease, it is unclear whether one causes the
other or if both may arise from some common mechanism. A recent study has
indicated that depressive symptoms are an early feature of Parkinson's disease,
preceding the characteristic movement problems seen in Parkinson's such as tremor
and rigid muscles. Therefore, people with signs of depression who start to develop
movement problems should be promptly evaluated to rule out a diagnosis of
Parkinson's disease.
Dementia and depression may be difficult to differentiate, as people with either
disorder are frequently passive or unresponsive, and they may appear slow, confused,
or forgetful. The Mini-Mental State Examination (MMSE) is useful to assess cognitive
skills in people with suspected dementia. (The MMSE examines orientation, memory,
and attention, as well as the ability to name objects, follow verbal and written
commands, write a sentence spontaneously, and copy a complex shape.) Early and
accurate diagnosis of dementia is important for patients and their families because it
allows early treatment of symptoms. For people with other progressive dementia, early
diagnosis may allow them to plan for the future while they can still help to make
decisions. These people also may benefit from drug treatment.
5

Risk Factors for Completed Suicide:


Sex: The person most likely to succeed in a suicidal attempt is a white male. While
females are more likely to attempt suicide, males are more likely to complete one.
Age: Although overall suicidal behaviors do not increase with age, rates of completed
suicide do increase with age.

• Elderly persons attempting suicide are also more likely to be widows/widowers, live
alone, perceive their health status to be poor, experience poor sleep quality, lack
a confidante, and experience stressful life events.
• Importantly, approximately 75 percent of elderly persons who commit suicide had
visited a primary care physician within the preceding month, but their
symptoms were not recognized or treated, underscoring that physicians must
be tuned into the signs and symptoms of depression and risks for suicide. Drug
overdose is the most common means of suicide on the elderly, making the
safety of medications chosen to treat the condition important.
Previous attempts: Having previously attempted suicide is a risk factor for attempting
suicide again.

Being a member of an ethnic minority does not make you more likely to attempt
suicide.
Poverty by itself is not a risk factor.

Major Depression Diagnostic Criteria


• For a diagnosis of major depression, the patient must have at least five of the
following nine criteria for a minimum of two weeks.
• A least one of the symptoms must be either (1) depressed mood or (2) loss of
interest or pleasure.

Depressed Mood


(The eight remaining criteria can be remembered using the mnemonic SIG E
CAPS):


Sleep: Insomnia or hypersomnia nearly every day.


Interest (loss of): Anhedonia (loss of interest or enjoyment) in usual activities.


Guilt: Feelings of worthlessness or excessive or inappropriate guilt (which may be
delusional) nearly every day (not merely self-reproach or guilt about being sick).


Energy (decreased): Fatigue or loss of energy nearly every day.



Concentration (decreased, or crying): Diminished ability to think or concentrate, or


indecisiveness, nearly every day (either by subjective account or as observed by others).


6

Appetite (increased or decreased): or significant weight loss when not dieting or weight
gain (e.g., a change of more than 5% of body weight in a month).


Psychomotor retardation: Psychomotor agitation or retardation nearly every day
(observable by others, not merely subjective feelings of restlessness or being slowed
down).


Suicidal ideation: Recurrent thoughts of death (not just fear of dying), recurrent suicidal
ideation without a specific plan, or a suicide attempt or a specific plan for committing
suicide.

Major Depressive Disorder vs Bereavement


The diagnosis of Major Depressive Disorder is generally not given unless the
symptoms are still present two months after the loss. However, the presence of
certain symptoms that are not characteristic of a "normal" grief reaction may be helpful
in differentiating bereavement from a Major Depressive Episode. These include:
• Guilt about things other than actions taken or not taken by the survivor at the time of
the death;
• Thoughts of death other than the survivor feeling that he or she would be better off
dead or should have died with the deceased person;
• Morbid preoccupation with worthlessness;
• Marked psychomotor retardation;
• Prolonged and marked functional impairment; and
• Hallucinatory experiences other than thinking that he or she hears the voice of, or
transiently sees the image of, the deceased person.

Risk factors for Late-life depression


Risk factors for late-life depression include:
• Female sex
• Social isolation
• Widowed, divorced, or separated marital status
• Lower socioeconomic status
• Comorbid general medical conditions, e.g. stroke, heart disease & cancer.
• Uncontrolled pain
• Insomnia
• Functional impairment
• Cognitive impairment

Depression in the Elderly


Depression is a very serious disease in the elderly:
• Depression increases the risk of disabilities in mobility and the activities of daily living
by about 70% over the course of 6 years.
7

• Alcohol and drug abuse are very common comorbidities complicating depression.
Completed suicide is more common in older depressed patients.

Assessing Severity of Suicidal Ideation


A tool used to assess whether a patient is seriously contemplating suicide is the SAD
PERSONS scale:
Sex (male)
Age (< 19 or > 45)

Depression, diagnosis of



Previous attempt(s)

Ethanol or other substance abuse

Rational thinking impaired (psychosis, delusions, hallucinations)

Social supports lacking

Organized plan for suicide

No significant other

Sickness (physical illness)

One point is scored for each factor present.


• A score of 4 to 6 suggests outpatient treatment is an appropriate clinical action
• A score of 7 to 10 suggests hospitalization is warranted

It can also be helpful to talk with the patient's friends and family, maintaining
appropriate confidentiality, although the professional should not agree to withhold
information if a patient is truly suicidal.

Antidepressant Medications
Most antidepressants work by improving the levels of the neurotransmitters
norepinephrine (NE), serotonin (5HT), and dopamine (DA). There are four major
classes of antidepressants:
Class Mechanism Examples
Selective serotonin Selectively block reuptake of serotonin, Citalopram (Celexa) 
Fluoxetine (Proza
reuptake inhibitors (SSRIs) potentiating serotonin's effect on the 
Fluvoxamine (Luvox)

post-synaptic neuron Paroxetine (Paxil)

Sertraline (Zoloft)

Escitalopram (Lexapro)
Tricyclic antidepressants Block reuptake of norepinephrine and Nortriptyline (Pamelor)
(TCAs) serotonin, potentiating their effects on Amitriptyline
8

the post-synaptic neuron Clomipramine (Anafranil)


Doxepin (Sinequan)
Monoamine oxidase (MAO) Block pre-synaptic catabolism of Phenelzine (Nardil)

inhibitors norepinephrine and serotonin (rarely used Tranylcypromine (Parnate)
today)
Others Serotonin and norepinephrine reuptake Venlafaxine (Effexor)
inhibitors Duloxetine (Cymbalta)
Norepinephrine and Bupropion (Wellbutrin)
dopamine reuptake
inhibitors
Serotonin antagonist and N efazodone (Serzone) and Trazodone
reuptake inhibitors (Desyrel)
Norepinephrine and Mirtazapine (Remeron)
serotonin antagonist,
antihistaminic effects
Serotonin partial agonist VIIBRYD (Vilazodone HCl)
and reuptake inhibitor

Side Effects of SSRI/SNRIs


Common side effects of SSRI/SNRIs include:
• Headaches
• Sleep disturbances--drowsiness and, less frequently, insomnia
• Gastrointestinal problems such as nausea and diarrhea
• Sexual dysfunction

They can also cause:


• Hyponatremia, due to the syndrome of inappropriate secretion of antidiuretic
hormone (SIADH)
• Serotonin syndrome (lethargy, restlessness, hypertonicity, rhabdomyolysis, renal
failure, and possible death)
• Increased risk of gastrointestinal bleeding
In the elderly you also have to be concerned about an increased risk for falls with these
medications, and recent studies show that they might have adverse effects on bone
density.

Older antidepressants such as TCAs can cause arrhythmias. Citalopram and


Escitalopram can cause QT interval prolongation at higher doses, especially in the face
of hypokalemia and hypomagnesemia or when combined with other medication that
have this same effect. Reports of symptomatc arrythmia is uncommon.
9

Often patients with depression will present with arthralgias and myalgias,
but SSRI/SNRIs do not cause arthralgias.

When treating patients with major depression disorder, a biopsychosocial approach


should be considered. "Bio" refers to pharmacotherapy; "psycho" refers to
psychotherapy; and "social" refers to the identification of life stressors.

Medication:
In a first episode of depression, it's usually recommended that the patient take the
medication for 9-12 months, as stopping any sooner runs a high risk for recurrence.
Recurrent episodes of depression are treated for two to three years. With multiple
recurrences and, in the elderly, who experience increased rates of recurrence,
continuous therapy should be considered.
SSRIs, such as sertraline, and SNRIs are generally considered safe and effective drugs
for depression. They have lower rates of side effects compared to the older tricyclics
and, unlike the tricyclics, have little risk in overdose. A tricyclic such as amitriptyline
would not be a first-line approach.
Psychotherapy:
Psychotherapy, most notably cognitive behavior therapy and interpersonal therapy,
have been found as effective as psychotropic medications. It can be especially useful for
patients who want to avoid medication.
Exercise:
Trials of mixed exercise indicated a small but statistically significant positive effect
favoring exercise for the treatment of mild to moderate depression and, similarly to
combining psychotherapy and medication, may have an additive effect when used
in combination simultaneously with other modalities.
Avoidance of other substances:
Additionally, avoidance of recreational drug and excessive alcohol use is a necessary
part of any treatment regimen.
ECT:
While ECT is not an appropriate treatment for an initial episode of major depression,
it is a safe and effective therapy that can be useful in patients with psychotic depression
or severe nonpsychotic depression unresponsive to medications or psychotherapy.

Antidepressant Discontinuation Syndrome


This phenomenon occurs in approximately 20 percent of patients after abrupt
discontinuation of an antidepressant medication that was taken for at least six
weeks. Typical symptoms of this syndrome include:
• flu-like symptoms
• insomnia
10

• nausea
• imbalance
• sensory disturbances, and
• hyperarousal.

These symptoms usually are mild, last one to two weeks, and are rapidly extinguished
with reinstitution of antidepressant medication. Antidepressant discontinuation
syndrome is more likely with a longer duration of treatment and a shorter half-life of
the treatment drug.

Profiles
Drug Comments
Fluoxetine • Unusually long half life (2-4 days), so effects can last for weeks after
(Prozac) discontinuation.
• Most problematic (but uncommon) side effects include agitation, motor
restlessness, decreased libido in women, and insomnia.
Sertraline • In addition to being a frequently used SSRI in pregnancy and breastfeeding,
(Zoloft) approved specifically for obsessive-compulsive, panic, and posttraumatic stres
disorders.
• More gastrointestinal side effects than the other SSRIs.
Paroxetine • Strong antianxiety effects.
(Paxil) • Side effects can include significant weight gain, impotence, sedation, and
constipation.
• Due to its short half-life, paroxetine is most likely of all the SSRIs to cause
antidepressant discontinuation syndrome.
Fluvoxamine • Particularly useful in obsessive-compulsive disorder.
(Luvox) • Greater frequency of emesis compared to other SSRIs.
Citalopram • Most common side effects include nausea, dry mouth, and somnolence.
(Celexa) • maximum recommended dose: 20 mg per day for patients 60 years of age due to
concerns of QT interval prolongation.
Escitalopram • Approved specifically for Generalized Anxiety Disorder.
(Lexapro) • Overall, fewer side effects than citalopram.

Evaluation of Fatigue or Depression


• A complete metabolic panel screens for electrolyte, renal, and hepatic problems
• A TSH can detect hypothyroidism
• A CBC will show anemia and vitamin deficiencies

• A urinalysis (B) is unlikely to be useful unless the depression or fatigue is of recent


11

onset and there is suspicion of infection.


• A brain CT scan (C) is unlikely to yield results in the absence of obvious neurologic
changes.
• A chest x-ray (E) is unlikely to add anything in the absence of specific symptoms such
as cough or shortness of breath.

Medications linked to depression: Clonidine, digitalis, hydralazine, propranolol,


thiazide, azathioprine, cisplatin, doxorubicin, vincristine, bromocriptine, levodopa,
fluphenazine, Haldol, barbiturates, benzos, ethanol, anticonvulsants, isoniazid,
tetracycline, caffeine, cocaine withdrawal, methylphenidate, steroid.

Depression in Hispanics
Due to factors such as economics, culture, and differences in presentation, Hispanics
have their depression identified less frequently than non-Hispanic whites. This holds
true in some other ethnic groups as well, such as African-Americans.
Hispanic patients will more frequently present to a doctor for somatic
complaints such as myalgias or fatigue, rather than with stated mood related
complaints.
U.S.-born Hispanics experience depression at similar rates to other ethnic groups.
Rates of depression in immigrant Hispanics are up to 50% lower than U.S.-born
Hispanics.
Psychosis is no more common in Hispanics than other groups, but symptoms of
perceptual distortion such as hearing noises or seeing shadows (known as celajes) are
more common and must be differentiated from psychotic hallucinations.
Hispanics and other ethnic and economic minorities are less likely to receive adequate
therapies.

• The ESFT model is a method of eliciting the patient's perspective. Evaluating in these areas
can increase patient satisfaction, adherence to recommended therapy, and health
outcomes.
• Explanatory model of the illness from the patient's perspective.
• Social and financial barriers to adherence.
• Fears and concerns about the medication or its potential side effects.
• Therapeutic contracting and playback adherence.

Elder Abuse
Early research indicates the following risk factors for abuse:
• Dementia.
• Shared living situation of elder and abuser (except in financial abuse).
• Caregiver substance abuse or mental illness.
• Heavy dependence of caregiver on elder. Surprisingly, the degree of an elder's
12

dependency and the resulting stress has not been found to predict abuse.
• Social isolation of the elder from people other than the abuser.

An estimated 2-10% of the elderly will at some point experience abuse, which can fall into
one of five categories:
• Physical abuse, which includes acts done with the intention of causing physical pain
or
injury.
• Psychological abuse, defined as acts done with the intention of causing emotional pain or
injury.
• Sexual assault.
• Material exploitation, involving the appropriation of the older person's money or property.
• Neglect.

Adherence to Antidepressant Medication in the Elderly


Providers note adherence to depression treatment in older adults occurs only about
half the time. The reasons are understandable and include:
• Inability to afford the medication
• Concerns about side effects
• Worry about the stigma of the diagnosis
• Not understanding how to take the medication properly

The important thing is to not blame the patient, but to educate her/him about the
recommendations, allowing the patient to ask questions and fully express any
concerns.

Case 9: 50F with palpitations (Panic attack, CAD)

Independent Risk Factors For Coronary Artery Disease


There are many risk factors that have been independently associated with CHD.
Fortunately, most of a person's risk for CHD and for stroke (together called
atherosclerotic cardiovascular disease, or ASCVD) can be determined by a limited set
of major risk factors. Other minor risk factors are only helpful if they adjust a patient's
risk category from that determined by the major risk factors.
In its 2013 guidelines on ASCVD risk assessment, the ACA/AHA task force delineated
these major risk factors and a few additional risk factors that may be helpful in further
categorizing a patient's risk of ASCVD. Among others, low HDL-c (<40), diabetes,
and current smoking are considered major risk factors.

Note: The mnemonic PQRST can be used to gather information about any chief
complaint. Chest discomfort provoked by exertion is a classic symptom of angina. It is
important to fully characterize exertional chest discomfort when it is identified.
13

• Provocation / palliation: Other factors that may provoke angina include cold,
emotional stress, meals, or sexual intercourse.
• Quality: Anginal pain is often described as squeezing, tightness, or pressure "like an
elephant sitting on my chest" although descriptions can vary widely.
• Region / radiation: Anginal pain may radiate to the neck, throat, lower jaw, teeth,
upper extremity, or shoulder. A wide extension of chest pain radiation increases
the probability that it is due to myocardial infarction and radiation to both arms
is another strong predictor of acute myocardial infarction.
• Associated symptoms: Anginal pain is often associated with sweating, shortness of
breath, and nausea.

Differential for Palpitations in a 50-Year-Old Woman


More Likely Diagnoses:

Dysrhythmias
Frequently cause palpitations, though many patients with dysrhythmias may not report
palpitations
Valvular heart disease
Can cause palpitations. Examples: aortic insufficiency or stenosis, mitral valve prolapse,
atrial or ventricular septal defects, and congenital heart disease
Hyperthyroidism
Can cause sinus tachycardia, atrial fibrillation, and other kinds of supraventricular
tachycardias - all of which can cause palpitations
(Hypothyroidism can cause bradycardia but would not present with palpitations)
Coronary heart disease (CHD)
Palpitations may be included in an atypical presentation of CHD
Historical items can suggest a cardiac cause of palpitation:
- Duration of palpitations greater than five minutes
- Description of an irregular beat (for example, the patient can tap it out with their
fingers)
- Previous history of heart disease
- Male sex
History of palpitations affecting sleep or during work would increase the likelihood that
an arrhythmia is the cause of palpitations
Anxiety / panic disorder
Frequent cause of palpitations
In a prospective cohort study of 190 patients at a university medical center who
complained of palpitations:
- 31% had palpitations caused by anxiety or panic disorder
- 43% percent had palpitations due to cardiac causes
14

(40% dysrhythmias and 3% other cardiac causes)


- 6% due to prescription or recreational use
Vasomotor symptoms of the climacteric (menopause)
May cause palpitations in peri-menopausal women
Associated with heat intolerance and sweating during a hot flash
Anemia
Particularly if it is acute and associated with tachycardia may present in this fashion
Drugs
Including caffeine, alcohol, tobacco, street drugs such as cocaine as well as numerous
prescription drugs used to treat the cardiovascular system
Sympathomimetics, vasodilators, anticholinergic drugs or withdrawal from beta
blockers can cause palpitations

Cardiovascular
Check blood pressure, heart rate, and heart rhythm.
Assess central venous pulsations in the neck. Visible neck pulsations would increase
your concern for an arrhythmic cause of the palpitations.
Listen for murmurs.
- A midsystolic click with a crescendo-decrescendo systolic murmur would suggest
mitral valve prolapse, the most common structural heart abnormality presenting
with palpitations.
- A harsh holosystolic murmur at the left sternal border that increases with Valsalva
maneuver would suggest hypertrophic obstructive cardiomyopathy that may be
associated with atrial fibrillation as a cause of palpitations.
- A harsh holosystolic murmur that radiates into the carotid arteries would suggest
aortic stenosis that can cause palpitations and dizziness.
Pulmonary
Crackles would suggest congestive heart failure that can occur with rapid atrial
fibrillation and other dysrhythmias.
However, if CHF were present, you would expect dyspnea, orthopnea, and possibly
ankle edema to be part of the patient's history.
Endocrine
Examine thyroid gland for enlargement or mass.
Hematologic
Look for pallor in conjunctiva and hands. Pallor would suggest severe anemia.
Musculoskeletal
Palpate the chest wall for tenderness. If present, this might suggest a musculoskeletal
cause for the patient's chest discomfort.
Skin
15

Check for evidence of sweating. Diaphoresis can be associated with vasomotor


symptoms of menopause, anxiety, and hyperthyroidism.
Check for hair loss that might occur with hyperthyroidism.
Psychiatric
Note any mannerisms that would suggest anxiety such as: fidgeting, hand wringing,
difficulty making eye contact, worried facies.

4 "P's": Pain Characteristics That Decrease the Likelihood of ACS


• Pleuritic -- worsened by respiration and may be exacerbated when lying down.
Causes of pleuritic chest pain include pulmonary embolism, pneumothorax, viral
or idiopathic pleurisy, pneumonia, and pleuropericarditis.
• Pulsating.
• Positional -- could be pericarditis, which typically improves with sitting up and
leaning forward; pleuritic chest pain, which may be exacerbated when lying
down; or musculoskeletal pain, which body position or movement may
exacerbate.
• Reproduced by Palpation -- musculoskeletal pain.
Another characteristic that speaks against pain of CHD is stabbing pain, but it is not a
"P"!

Pressure-like pain is only associated with positive likelihood ratios of 1 to 2 and is less
helpful.
Coronary Heart Disease in Women
In a landmark article on women and prodromal symptoms of myocardial infarction,
McSweeny and colleagues noted that 95% of women reported prodromal symptoms,
but only 29.7% reported chest discomfort.
Prodromal symptoms of ACS in women may include:
• fatigue
• dyspnea
• neck and jaw pain
• palpitations
• cough
• nausea and vomiting
• indigestion
• back pain
• dizziness
• numbness
These symptoms are reported less frequently by men prior to myocardial infarction

• Although CHD deaths in the US have declined in men, the number of deaths due to
16

CHD in women has increased. Although men are still diagnosed more often
with CHD, the proportion of women presenting with clinically and
angiographically significant disease has increased over the last 20 years.
Women are less aggressively treated than men and have worse outcomes. The fact
that women are usually older at presentation than men and that women tend to wait
longer before seeking treatment may be part of the reason. Women are also less likely
to participate in cardiac rehabilitation. Unfortunately, one study showed that in
ambulatory care setting, women were less likely to be treated with a beta-blocker,
aspirin or a statin even after having a heart attack. Effective early diagnostic strategies
for diagnosing CHD earlier in women are critical, as 40 percent of initial cardiac events
in women are fatal.

Erikson’s stages of human development.

With regard to coronary heart disease there are several differences between men and
women. CHD incidence in women lags 10 to 15 years behind that of their male
counterparts, with women not catching up until about their seventh decade. Under-
recognition and under-diagnosis of heart disease in women has been shown to
contribute to high mortality rates in women. Up to 40 percent of initial cardiac events in
women at risk are fatal. Clearly, we need more effective methods for diagnosis.
Unfortunately, there are not many studies with large numbers of women to clearly
determine the value of detecting CHD in women.

 Evidence has shown a decreased
diagnostic accuracy and higher false positive rates for noninvasive testing in women
compared to men. One of the theorized reasons is that women tend to have single
vessel disease.

 Another factor is based in probability statistics. The post-test
likelihood of disease is heavily influenced by a patient's pretest risk. Referral of low risk
women will yield a high rate of false positive stress tests. The sensitivity and specificity
of exercise treadmill testing in women is 70% and 61%, respectively. 

Risk factors and
symptom classifications are based primarily on studies in men. However, the addition
of other parameters, such as exercise capacity and heart rate changes, with the
traditional evaluation of ST-segment changes, improves the prognostic accuracy of the
exercise treadmill in women. Studies using The Duke Treadmill Score have shown
improvement in the predictive value of the stress test. Given the confusion of
diagnostic accuracy of exercise stress testing in women, the undetected prevalence of
CHD, and the atypical symptoms with which women present may argue for the use of
radionucleotide stress testing in women.

Aspirin for Primary Prevention of CHD


• The USPSTF recommends initiating aspirin therapy in men age 45 to 79 years to
reduce the risk of myocardial infarction. In women age 55 to 79, the USPSTF
17

advises taking aspirin to reduce the risk of ischemic stroke. For both men and
women, the benefit of decreased risk from those outcomes must be weighed
against an increased threat of gastrointestinal hemorrhage.
• The European Guidelines on Cardiovascular Disease Prevention in Clinical Practice
states that aspirin cannot be recommended for primary prevention in
individuals without overt cardiovascular or cerebrovascular disease due to
increased risk of major bleeding. These guidelines do specify that under certain
circumstances of high cardiovascular risk or overt cardiovascular/cerebrovascular
disease, aspirin prescription may be considered.

Case 10: 45M with low back pain (disc herniation)

Most Common Causes of Back Pain


There are three major categories of back pain: mechanical, visceral, and non-
mechanical.

Mechanical
• 97% of back pain
• no primary inflammatory or neoplastic cause

Visceral
• 2% of back pain
• no primary involvement of the spine, usually from internal organs

Non-mechanical
• 1% of back pain
• other

The three most common causes of back pain are all mechanical:
1. lumbar strain/sprain - 70%
2. age-related degenerative joint changes in the disks and facets - 10%.
3. herniated disc - 4%

Less common causes of mechanical back pain:


• osteoporotic fracture - 4%
• spinal stenosis - 3%

Uncommon causes of back pain:


• Pyelonephritis, a visceral cause, accounts for 0.4% of back pain.
Kidney stones are an unusual cause of Lower back pain.
18

Red Flags For Serious Illness or Neurologic Impairment with Back Pain
• fever
• unexplained weight loss
• pain at night
• bowel or bladder incontinence
• neurologic symptoms

Risk Factors for Low Back Pain


• Prolonged sitting, with truck driving having the highest rate of LBP, followed by desk
jobs.
• Deconditioning.
• Sub-optimal lifting and carrying habits.
• Repetitive bending and lifting.
• Spondylolysis, disc-space narrowing, spinal instability, and spina bifida occulta.
• Obesity is a possible risk factor, but the evidence is limited and inconsistent.

Recommended Low Back Pain History


1. History of present illness.
• What is the location of the pain? Is it upper, middle or lower back?
• What is the duration of the pain or how long ago did it start? Is it getting worse or
better? Does the pain radiates? Pain that radiates below the knee- more
consistent with sciatica; pain around the buttock-more consistent with
lumbar strain.
• What is the severity of the pain? Use a pain scale of 1-10 to make the severity
somewhat more objective. Intensity of the pain
• What is the quality of the pain? Is it achy, or sharp, or dull, or throbbing?
• Is the pain constant or remitting? Is it present at night or at rest?
• Are there associated symptoms? Does the patient have weakness or numbness or
tingling?
• Are there aggravating or alleviating factors? Aggravating circumstances (active vs.
passive motion, day vs. night). Valsalva can increase pain from a herniated
disk.
• Alleviating circumstances (medication, positioning-sitting, lying, standing) What has
the patient tried to relieve the problem (what worked, what didn't)
• Any history of similar problems?

2. Pertinent past history. Recent illnesses, history of recent trauma or injury, patient's
occupation, previous history of back injury, cancer, or DM. (Fatigue is a nonspecific
finding which may not help you to narrow your differential diagnosis.)
19

3. Review of systems. In order to narrow your differential diagnosis for the patient's
problem, a review of systems, focused on pertinent positives and negatives is
important.
• Neurologic symptoms (saddle anesthesia, lower extremity numbness, tingling, muscle
weakness particularly in the lower extremities, fecal incontinence)
• Urinary symptoms (urinary incontinence, hesitancy, frequency, dysuria)
• Gastrointestinal symptoms (nausea, vomiting)
• Constitutional symptoms (fever, unexplained weight loss)

4. Current medications and allergies

Differential for Low Back Pain


The most appropriate diagnoses on your differential include:
Lumbar strain, disc herniation, spinal stenosis, and degenerative arthritis.

Pain worse with movement and sitting is suggestive of a mechanical cause of back pain,
such as a lumbar strain, disc herniation, or degenerative arthritis.
Pain radiating down the leg and numbness indicate nerve involvement, such as in
disc herniation.
Pain that improves with the supine position suggests spinal stenosis and disc
herniation.

The following diagnoses are less likely:

Spinal fracture: Not likely without history of trauma.


Cauda equina syndrome: Should always be considered due to the seriousness of the
consequences. Occurs when a large mass effect (such as an acute disc herniation or a
tumor) compresses the cauda equina, causing pain radiating down the leg and
numbness of the leg. True emergency. Decompression should be performed within 72
hours to avoid permanent neurologic deficits. Low on the differential if the patient
denies problem with bowel or bladder control.
Pyelonephritis: Unlikely with lack of fever and urinary symptoms.
Malignancy: Important consideration. A very serious, although uncommon, cause of
back pain. Unlikely without a history of cancer. Back pain due to malignancy is localized
to the affected bones, it is a dull, throbbing pain that progresses slowly, and it increases
with recumbency or cough. More commonly seen in patients over 50.
Ankylosing spondylitis: Chronic, painful, inflammatory arthritis primarily affecting
the spine and sacroiliac joints, causing eventual fusion of the spine.
20

Often seen in patients 15-40 years old, associated with morning stiffness and achiness
over the sacroiliac joint and lumbar spine.
Spondylolisthesis: Anterior displacement of a vertebra or the vertebral column in
relation to the vertebrae below. Can occur at any age. Causes aching back and posterior
thigh discomfort that increases with activity or bending.
Prostatitis: Can cause referred LBP in men. (Pelvic inflammatory disease and
endometriosis in women can cause referred LBP). Expect to find evidence of infection
in the history.
Pancreatitis: Pancreatitis and other gastrointestinal diseases such as cholecystitis and
ulcers can cause LBP via visceral pain. Usually associated with other abdominal
symptoms.

Physical Exam for Back Pain - Standing


Throughout the whole exam make certain to note how your patient is sitting, standing,
and walking in general, asking yourself 'What is his degree of impairment?' and 'How
uncomfortable is he?'
I. Inspection: Look at posture, contour and symmetry
• Check for lordosis
• Check for kyphosis
• Check for scoliosis
Slight scoliosis may be more easily visualized during lumbar flexion. This is performed
by having the patient stand with their feet and hands together, like they are about to
dive off a diving board, bending forward toward their toes. Look out across the back to
see if the shoulders are level.
II. Palpation: Check for any tenderness, tightness, rope-like tension, or inflammation in
the paraspinous muscles or tenderness over bony prominences. This procedure checks
for muscle spasm, vertebral fracture, or infection.
III. Range of Motion (ROM):
• Lumbar Flexion (normal is 90 degrees): This is the best measure of spine mobility.
Restriction and pain during flexion are suggestive of herniation,
osteoarthritis, or muscle spasm.
• Lumbar Extension (normal is 15 degrees): Pain with extension is suggestive of
degenerative disease or spinal stenosis.
• Lateral motion (normal is 45 degrees): Most patients should be able to touch the
proximal fibular head of the knee. Pain on the same side as bending is suggestive
of bone pathology, such as osteoarthritis or neural compression. Pain on the
opposite side of bending is suggestive of a muscle strain.
• Range of motion may be varied due to the patient's age and body habitus
IV. Gait: Ask the patient to walk on heels and toes. Expect normal gait, even with disc
herniation.
21

• Difficulty with heel walk is associated with L5 disc herniation


• Difficulty with toe walk is associated with S1 disc herniation
V. Stoop Test: Have the patient go from a standing to squatting position.
In patients with central spinal stenosis, squatting will reduce the pain. However, asking
the patient to run is not part of a back exam and may cause discomfort to the patient
who is already in pain.

Physical Exam for Back Pain - Seated Position


Overview of the Neurologic Exam

Nerve Impingement Syndromes


• Check for costovertebral angle (CVA) tenderness, a sign suggesting pyelonephritis.
• Modified version of the straight leg raise (SLR) test
• While continuing to talk to the patient, raise each leg by extending the knee from 90
degrees to straight. If the pain is functional, the action is possible without
difficulty. If the pain is due to structural disease, the patient will instinctively
exhibit the "tripod sign" by leaning backward and supporting himself with his
outstretched arms on the exam table.
• (The unmodified version of the straight leg raise (SLR) test is done in the next section
of the exam with the patient supine.)
• Neurological exam
Check reflexes, muscle strength, and sensation of the lower extremities. Focus on the
L4, L5, and S1 nerve roots because most neuropathic back pain is due to impingement
of these. Therefore, check the patellar and Achilles reflexes. Check muscle strength for
hip flexion, abduction, and adduction; knee extension and flexion; as well as ankle
dorsiflexion and plantar flexion. Also, test for sharp and light touch along the
dermatomal distribution of the great toe (L5), lateral malleolus and posterolateral foot
(S1).

Physical Exam for Back Pain - Supine


I. Abdominal Exam
• Auscultation: Check for abdominal bruit, looking for abdominal aortic aneurysm.
• Palpitation: Check for abdominal tenderness (on all patients, not just female
patients), pelvic tenderness (PID), pulsatile mass, unequal femoral/brachial
pulses (abdominal aortic aneurysm), or any general tenderness indicating
visceral pathology.
II. Rectal Exam
• To be done only on patients with red flags or alarm symptoms, which we will discuss
later!
• Check for masses, bleeding, or abnormal rectal tone. Bleeding or rectal mass can
22

be signs of cancer with metastasis to the spine causing back pain. Decreased
tone can indicate disc herniation and/or cauda equina syndrome.
III. Passive Straight Leg Raise (SLR or Lasegue's sign)
• The normal leg can be raised 80 degrees.
If a patient only raises their leg <80 degrees, they have tight hamstrings or a
sciatic nerve problem.
To differentiate between tight hamstrings and a sciatic nerve problem, raise
the leg to the point of pain, lower slightly, then dorsiflex the foot. If there
is no pain with dorsiflexion, the patient's hamstrings are tight.
• The test is positive if pain radiates down the posterior/lateral thigh past the knee.
This radiation indicates stretching of the nerve roots (specifically S1 or L5) over a
herniated disc.
• This pain will most likely occur between 40 and 70 degrees. Pain earlier than 30
degrees is suggestive of malingering.
• Pain in the opposite leg during a straight leg raise is suggestive of root compression
due to central disc herniation.
• When compared to MRI, the straight leg raise test has a sensitivity of 0.36 and
specificity of 0.74
• The ipsilateral straight leg raise test has a sensitivity of 0.80 and a specificity of about
0.40. Thus, a negative test makes a herniated disc unlikely, but a positive test is
non-specific.

IV. Crossed Leg Raise: asymptomatic leg is raised


Test is positive if pain is increased in the contralateral leg; this correlates with the
degree of disc herniation. Such results imply a large central herniation.
Cross SLR test is much less sensitive (0.25) but is highly specific (about 0.90). Thus, a
negative test is nonspecific, but a positive test is virtually diagnostic of disc herniation.

V. FABER Test: Flexion, Abduction, and External Rotation


• The Faber test looks for pathology of the hip joint or sacrum (sacroiliac pain from
sacroiliitis).
• The test is performed by flexing the hip and placing the foot of the tested leg on the
opposite knee. Pressure is then placed on the tested knee while stabilizing the
opposite hip.
• The test is positive if there is pain at the hip or sacral joint or if the leg cannot lower to
the point of being parallel to the opposite leg.
The FABER test should be done on all patients suspected of having sacroiliac pain, not
just in the elderly patients. Sacroiliitis can occur in the young population as well.

VI. Pelvic Compression Test


23

• Test is performed by forcibly pressing together the hips


• A positive test elicits pain in the sacroiliac joint.

VII. Muscle Atrophy: of quadriceps and calf muscles; lack of atrophy, despite patient's
complaints of long-term weakness, suggests malingering.

Symptoms of Disc Herniation


When disc herniation is suspected, a very important historical point is the position of
comfort or worsening of symptoms. Classically, disc herniation is associated
with exacerbation when sitting or bending; and relief while lying or standing. Other
symptoms of disc herniation include:
• increased pain with coughing and sneezing
• pain radiating down the leg and sometimes the foot
• paresthesias
• muscle weakness, such as foot drop

Red Flags for Serious Underlying Causes of Back Pain


While the majority of back pain has a benign course and resolves within a month, a
small number of cases are associated with serious underlying pathology. Timely
treatment of these conditions is important to avoid serious consequences. Indications
for early diagnostic testing such as x-rays and other imaging and referral are patients
with progressive neurological deficits, patients not responding to conservative
treatment (which we will discuss in a minute), and patients with red flags signaling
serious medical conditions such as fracture, cancer, infection, and cauda equina
syndrome. Knowing this would also help guide the evaluation and treatment of the
back pain.

Red Flags...

Cancer
1. History of cancer
2. Unexplained weight loss >10 kg within 6 months
3. Age over 50 years or under 17 years old
4. Failure to improve with therapy
5. Pain persists for more than 4 to 6 weeks
6. Night pain or pain at rest

Infection
1. Persistent fever (temperature over 100.4 F)
24

2. History of intravenous drug abuse


3. Recent bacterial infection, particularly bacteremia (UTI, cellulitis, pneumonia)
4. Immunocompromised states (chronic steroid use, diabetes, HIV)

Cauda Equina Syndrome


1. Urinary incontinence or retention
2. Saddle anesthesia
3. Anal sphincter tone decreased or fecal incontinence
4. Bilateral lower extremity weakness or numbness
5. Progressive neurologic deficits

Significant Herniated Nucleus Pulposus


1. Major muscle weakness (strength 3 of 5 or less)
2. Foot drop
Vertebral Fracture
1. Prolonged use of corticosteroids
2. Mild trauma over age 50 years
3. Age greater than 70 years
4. History of osteoporosis
5. Recent significant trauma at any age (car accident, fall from substantial height)
6. Previous vertebral fracture

Conservative Therapy for Acute Low Back Pain


Conservative therapy for acute low back pain includes:
• Pharmacologic therapy (aspirin/NSAID and/or muscle relaxants)
• Local therapy (heat/cold). Learn more about local therapy.

Strict bed rest has not been shown to be beneficial. Patients should be encouraged to
resume normal activities as soon as they are able to.
Referral to a surgeon or advanced imaging, such as MRI/CT scans, should be
entertained if back pain is not better in four to six weeks or if progression of neurologic
deficits is demonstrated.

Acute Low Back Pain Prognosis


Most cases of low back pain are acute in onset and resolution, with 90% resolving
within one month and only 5% remain disabled longer than three months.
For patients who are out of work greater than six months, there is only 50% chance of
them returning to work; this drops to almost zero chance if greater than two years.
Patients who are older (>45) and patients who have psychosocial stress take longer to
recover.
25

Recurrence rate for back pain is high at 35 to 75%.

Treatment After Adequate Trial of Conservative Therapy


If a patient has been in pain for five weeks with progression of neurological deficit (such
as absent reflex at the ankles) and poor pain control, it is reasonable to refer him to a
spine surgeon for surgical consultation or a pain clinic for possible epidural steroid
injection.
If the patient doesn't have any red flags, continuation of conservative treatment is also
an option. However, if the patient has already been getting PT, more PT is not likely to
help.
There is limited evidence in the use of acupuncture in the treatment of LBP, with some
studies showing that it is not beneficial.

Case 11: 74F with knee pain (Osteoarthritis)


Differential Diagnosis for Knee Pain
Diagnosis Location of History of trauma Fever Special notes
Systemic
pain symptoms
Patellofemoral Anterior knee No; overuse injury No Typically presents in
Pain Syndrome pain more likely women as "theater sign"
(chondromalaci - mild to moderate
a patellae) anterior knee pain,
worse after prolonged
sitting
Iliotibial Band Lateral knee No; overuse injury No No effusion
Tendonitis pain more likely (repetitive
knee flexion) Pain aggravated with
activity
Anterior General knee Yes; noncontact No Moderate to severe
Cruciate pain deceleration forces joint effusion
Ligament
Sprain Swelling within two
hours of "pop"
Medial Medial joint Yes; misstep or No Immediate onset of
Collateral line pain collision pain/swelling after
Ligament trauma
Sprain
Lateral Lateral joint Yes; varus stress No Immediate onset of
Collateral line pain lateral knee pain
Ligament
Sprain Less common than
26

medial collateral
ligament sprain
Meniscal Tear Medial or Yes; sudden twisting No Can occur with
lateral joint injury chronic degenerative
line process

Mild effusion

Possible atrophy of
the vastus medialis
obliquus portion of
the quadriceps

Catching/locking of
the knee

Can have positive


McMurray test
Septic arthritis Generalized No Yes Abrupt onset of pain
extreme pain and swelling
with any Elevated
movement WBC Arthrocentesis with
turbid synovial fluid
Elevated
ESR
(usually
>50
mm/hr)
Osteoarthritis Generalized or Not acute trauma. No Chronic joint stiffness
joint line and pain
tenderness; Past history of trauma
pain can predispose to Crepitus on exam
aggrevated by developing
weight- osteoarthritis Mild or no joint effusion
bearing
activities,
relieved by rest
Gout / Extreme pain No No Acute pain and swelling
pseudogout with any without prior trauma
movement;
27

also painful to Arthrocentesis with


touch clear or slightly cloudy
synovial fluid

Negatively birefringent
rods in gout

Positively birefringent
rhomboids in
pseudogout

Popliteal Posterior No No Insidious onset of


(Baker's) Cyst popliteal area mild to moderate pain
in the popliteal area
of the knee

Most common
synnovial cyst of the
knee

Screening for Alcohol Use / Abuse


Many alcohol use / abuse screening tools exist. The CAGE questionnaire, though popular,
appears to have highly variable ability to screen for alcohol issues in an ambulatory setting. It is
less variable in hospitalized patients.
Have you ever felt you should Cut down on your drinking?
Have people Annoyed you by criticizing your drinking?
Have you ever felt bad or Guilty about your drinking?
Have you ever had a drink first thing in the morning to steady your nerves or to get rid of a hangover
(Eye opener)?

A two-question screening tool was shown in a prospective cohort study to be an effective tool
to identify alcohol and other drug abuse. 


1) "In the last year, have you ever drunk alcohol or used drugs more than you meant to?"
2) "Have you felt you wanted or needed to cut down on your drinking or drug use in the last
year?"
The Alcohol Use Disorders Identification Test (AUDIT) is a validated 10-question screening tool
that approaches 90% sensitivity and specificity. A PDF about the AUDIT can be found at The
World Health Organization's website.

Ligament Mechanism of History Physical Exam Special Tests


Strain Injury
28

Anterior Noncontact "Pop" heard Effusion, Lachman, Anterior Drawer Sign;


Cruciate Deceleration F decreased ROM excessive motion and lack of clear
orce endpoint is a positive test; also
compare to opposite knee
Medial Misstep or Immediate Medial joint line Valgus stress testing with pain at 30
Collateral collision that pain/swelling point tenderness degrees flexion; excessive motion
causes valgus medially and/or presence of pain indicates
stress potential strain or ligamentous instability
Lateral Varus stress Immediate lateral Lateral joint line Varus stress testing pain/instability
Collateral knee pain point tenderness at 30 degrees flexion; excessive motion
and/or presence of pain indicates
potential strain or ligamentous instability

McMurray test
Can assess the medial and lateral menisci, though it has low sensitivity and
specificity for diagnosing meniscal tears. The physician holds the patient's heel with
one hand and grasps the knee over the medial and lateral joint lines with the other
hand. The patient's knee is flexed as much as possible. The tibia is rotated either
internally (tests lateral meniscus) or externally (tests medial meniscus) as the knee
is extended to about 90 degrees. A varus stress (lateral meniscus) or valgus stress
(medial meniscus) is applied across the knee joint while the knee is being extended. The
test is positive if a clunk or click is felt, or if testing causes reproducible knee pain.

Knee Exam:
Have the patient put on a gown, as it is important to be able to fully examine and
compare the painful knee and the non-painful knee.
Observe the patient walking and climbing onto the examination table.
Inspect both legs for erythema, edema, bruising, or atrophy of the quadriceps.
Palpate the knee joints, feeling for warmth, effusion, and point tenderness. Pay
particular attention to the patella, tibial tubercle, patellar tendon, quadriceps tendon,
anterolateral and anteromedial joint line, medial joint line, and lateral joint line.
Check knee range of motion by flexing and extending the knees (normal is 0 degrees
extension, and 135 degrees flexion).
Assess for tenderness and range of motion on hip exam.

Patellar subluxation is an incomplete or partial dislocation of the kneecap. The patella


repetitively subluxates and places strain on the medial restraints and excessive tension
on the patellofemoral joint. This is commonly caused by running and jumping sports
and activities that place large forces on the patellofemoral joint. A patellar
apprehension test helps detect patellar subluxation. The test is positive if there is pain
or a giving-way sensation when attempting to translate the patella laterally.

29


Tibial apophysitis, or an Osgood-Schlatter lesion, is inflammation of the tibial


tubercle at the site of the patellar tendon attachment. It is typically seen in
adolescents who recently went through a growth spurt. They present complaining of
anterior knee pain localized to the tibial tuberosity.

Key History for Knee Pain


Patient's age

Age can help narrow your differential diagnosis, as certain conditions are classically
seen in certain age groups.
• Children and adolescents who present with knee pain are likely to have patellar
subluxation, tibial apophysitis (Osgood-Schlatter), or patellar tendonitis). Please
see the Expert for additional information.
• Adults are more prone to patellofemoral syndrome (a clinical diagnosis of exclusion
for anterior knee pain), overuse syndromes (such as pes anserine bursitis),
traumatic injuries (ligamentous sprains - anterior cruciate, medial collateral,
lateral collateral - and meniscal tears) and inflammatory arthropaties, such as
rheumatoid arthritis, septic arthritis, and Reiter's syndrome.

Any previous history of trauma / injury to knee

Without a history of trauma, ligament injuries and meniscal tears are less likely.
These conditions often present with acute onset of pain following trauma.

Treatments that make it better

Analgesics can often help pain, regardless of the etiology. This is less helpful in
narrowing the differential diagnosis since it is so non-specific.

Actions that worsen the symptoms

Actions that worsen the symptoms are not typically helpful to narrow the differential
diagnosis. However, asking the patient to perform certain maneuvers can help to assess
the functioning of certain aspects of the knee as well as the extent the patient's pain
interferes with mobility. For example, the ability to squat is influenced by supporting
musculature, ligaments, and the knee joint. Impaired squatting ability could be
caused by effusion, knee arthritis, injury to the ligaments, etc., whereas the ability
to perform the duck waddle assesses the stability of the knee and effectively rules out
significant ligament instability, joint effusion, and significant damage to the meniscal
30

cartilage.

Acute versus insidious onset of pain

The time frame of the onset of pain does not usually help narrow the differential
significantly, although it can suggest whether to focus on acute (potentially traumatic)
injury as a possible cause in contrast to a chronic, systemic etiology of the condition.

Monoarticular joint involvement

Knowing which joint(s) are involved can be helpful, as certain conditions have an
affinity for particular joints. Gout (uric acid crystal deposition in the joint that causes
severe pain) often presents in the great toe, whereas rheumatoid arthritis typically
affects three or more joints, often including the hands and feet. Osteoarthritis
often affects the knees, hips, and back.

Differential for chronic knee pain


Top Five Diagnoses
• knee sprain
• osteoarthritis
• rheumatoid arthritis
• gout/pseudogout
• psoriatic arthritis
Knee sprain • Can be associated with a small effusion, and with pain that worsens with
• weight-bearing and activity.
• Unlikely without history of trauma or change in activities that would be expected
• with sprain.
Osteoarthriti • Usually the large joints are asymmetrically involved.
s • Can be monoarticular in young adults if due to trauma or a congenital defect, but more common
presents as polyarticular or generalized arthritis.
• Joint stiffness, if present, is typically worse after effort.
Rheumatoid • Can have associated subcutaneous nodules that are firm and nontender,
arthritis • and are located at pressure points.
• Joint stiffness for more than thirty minutes in the morning is common.
• Typically bilateral joint pain, involving three or more joints (in particular the
• hands or feet).
Gout • Can be associated with tophi which are visible or palpable nodules often located
• on the ears or in the soft tissue. Tophi can also form in the bones, joints, and cartilage.
• They are typically not painful, and take years to develop.
Psoriatic • Associated with psoriatic plaques, or thickened silvery scaly plaques located on
31

arthritis • the extensor surfaces.


• Psoriasis must be present to make this diagnosis, but the arthritis sometimes
• appears before the lesions (13-17% of cases), and lesions are present but
• undiagnosed in about 15% of cases.
• Usually oligoarthritis, meaning that it affects two to four joints, or polyarthritis,
• when five or more joints are involved.

Evaluation of Acute Knee Trauma - Ottawa Knee Rules



A validated set of criteria that can assist in determining which patients with knee pain
following trauma warrant an x-ray to rule of fracture. An x-ray series is only required for knee
injury patients with any of these findings:
• Age 55 or older
• Isolated tenderness of the patella (that is, no bone tenderness of the knee other than the
patella).
• Tenderness at the head of the fibula.
• Inability to flex to 90 degrees.
• Inability to bear weight both immediately and in the emergency department (4 steps;
unable to transfer weight twice onto each lower limb regardless of limping).

Evaluation of Knee Pain


Suspected infectious process causing knee pain. 
If concerned about septic arthritis or
an acute inflammatory arthropathy, check a Complete Blood Count (CBC) with
differential and erythrocyte sedimentation rate (ESR), though this test is non-
specific.

 Perform an arthrocentesis and send the fluid for cell count with differential,
glucose and protein, bacterial culture and sensitivity, and polarized light microscopy for
crystals. An arthrocentesis can also help differentiate between simple effusion and
hemarthrosis or occult osteochondral fracture.
• A simple joint effusion produces clear, straw-colored transudative fluid. This can
happen with osteoarthritis and degenerative meniscal injuries.
• Hemarthrosis is typically caused by a tear of the anterior cruciate ligament or a
fracture. A bloody knee aspirate can be associated with a knee sprain (i.e. ACL,
PCL) or acute meniscal tear. An osteochondral fracture causes hemarthrosis with
fat globules.

Suspected rheumatoid arthritis causing knee pain 
If considering RA, check
rheumatoid factor (RF) on blood work. This test is not very sensitive and only
moderately specific for rheumatoid arthritis. It would be more helpful to as a test to rule
out RF (if the test is negative), than to rule it in (if the test is positive).

Trauma causing knee pain


32

To evaluate knee pain following trauma, apply the Ottawa Knee Rules to decide
whether or not to order an x-ray.

Imaging to Evaluate Osteoarthritis


Indications
Imaging for knee pain in a patient with no preceding trauma may not be necessary
when the history and physical exam are consistent with osteoarthritis. However, if
there is any question as to the diagnosis, or you are interested in assessing the
severity/location of disease, an x-ray would not be unreasonable. An x-ray would also
be warranted if there is no improvement with initial conservative treatment.
Magnetic resonance imaging (MRI)
MRI would be preferred if locking, popping, or joint instability were of concern, to
detect meniscal or ligament damage. However, if an x-ray shows significant joint space
narrowing, an MRI would likely not be needed even if there were mechanical
symptoms, unless the patient fails to improve with conservative management.
Views
If a knee x-ray is desired to evaluate for osteoarthritis (OA), anteroposterior, lateral,
and standing weight-bearing views should be obtained. A Merchant's View can help
evaluate the patellofemoral joint. The Merchant's view is a "top" view of the knee
obtained with the knee bent at a 45-degree angle, showing the alignment of the
patella in the groove of the femur (throchlear groove).
Major radiographic features of OA
• Joint space narrowing
• Subchondral sclerosis (Hardening of tissue beneath the cartilage. In osteoarthritis,
there is increased periarticular bone density.)
• Osteophytes (Also known as bone spurs; bony projections arising from the joint.)
• Subchondral cysts (Fluid-filled sacs in the bone marrow.)
Interpretation
Knee x-rays are insensitive for detecting early OA and do not correlate well with the
degree of symptoms. When findings are present, patellofemoral and tibiofemoral joint
osteophytes correlate best with pain, and joint space narrowing best predicts
disease progression.

Osteoarthritis Epidemiology
Epidemiology

Osteoarthritis is very common among U.S. adults, and the leading cause of disability.
In fact, arthritis is expected to affect an estimated 67 million adults in the United States
by 2030. Findings from a National Health Interview Survey several years ago indicated
that an estimated 21.6% of the adult U.S. population (46.4 million persons) had doctor-
diagnosed arthritis. Both white and black races are at equal risk for the disease.
33

Initial management

Exercise has been shown to improve function and decrease pain in OA. Current
guidelines strongly recommend that patients with symptomatic knee OA participate
in an exercise program commensurate with their ability to participate; they do not
preferentially recommend aquatic or land-based exercise.

Treatment Results for Knee OA Evidence


Exercise • Specifically water- or land-based exercise, Reduce pain and A
aerobic walking, quadriceps strengthening, disability
resistance exercise, tai chi
Acetaminophen • First choice analgesic for both short and long- Reduce pain A
term treatment of mild to moderate pain
related to osteoarthritis because of its
tolerability and low side-effect profile.
• Dosing is up to 4 gm per day in divided doses,
though some recommend lower doses (2-3
gm/day in divided doses) if long-term use is
desired.
• Caution patients to be aware of coincident use of
other over-the-counter or prescription
medications that may contain
acetaminophen so that the maximum
combined daily dose does not exceed 4
gm.
• Little risk of nephrotoxicity, and hepatotoxicity
is a rare side effect if taken appropriately.
NSAIDS • Second choice to acetiminophen because of Reduce pain A
their association with gastrointestinal
side-effects, including gastritis.
• Especially true in the geriatric population
because of their increased risk of acute
gastrointestinal bleeding, even when used
in conjunction with antacids or proton pump
inhibitors.
• Risk of prolonged bleeding times, a potential
problem if the patient is at risk for falling.
Intra-articular • Should be considered if the knee joint is Short-term benefit A
corticosteroid inflamed, as evidenced by swelling and with few adverse
injections pain. effects
34

• No more than three injections per year, and no


more frequent than one injection per
month.
• Long-acting triamcinolone is typically preferred
over methylprednisolone, and 1 ml of
steroid should be combined with 3-4 ml
local anesthetic.
• 24 hours of immobilization following the
injection helps maximize the effects, but
prolonged rest should be avoided.
• Can reduce pain, limit the need for other
medications, and improve function.
• Fewer associated side effects than NSAIDs or
opiates.
Acupuncture • A 2006 systematic review showed that May provide some B
acupuncture is better than sham-control benefit
interventions for peripheral joint
osteoarthritis.
• Also shown to reduce pain and improve function
in patients with OA of the knee when used
as an adjunct treatment.
Glucosamine • A randomized controlled trial (RCT), followed by May provide some B
a meta-analysis failed to show a decrease in benefit for persons
pain or slowed progression of joint space with moderate to
narrowing related to glucosamine use. severe pain
• A small subset of participants in the GAIT study
with moderate-to-severe pain did show
statistically significant pain relief when
glucosamine was combined with
chondroitin sulfate as compared to placebo,
but more investigation was recommended
to confirm this finding because of the small
sample size. The same was not true of the
mild pain subset which showed no
improvement over placebo.
Chondroitin Does not decrease B
pain
S- As effective as B
adenosylmethioni NSAIDs in reducing
ne (SAM-e) pain and disability
35

Tramadol Older patients with B


(Ultram) moderate to severe
pain may experience
modest benefit; use is
limited by side effects

Non-Pharmacologic Recommendations for the Management of Knee Osteoarthritis


Strongly recommend that patients with knee OA:
• Participate in cardiovascular (aerobic) and/or resistance exercise - either aquatic or
land-based
• Lose weight (for persons who are overweight)

Conditionally recommend that patients with knee OA:


• Participate in self-management programs
• Receive manual therapy in combination with supervised exercise
• Receive psychosocial interventions
• Use medially directed patellar taping
• Wear medially wedged insoles if they have lateral compartment OA
• Wear laterally wedged subtalar strapped insoles if they have medial compartment
OA
• Be instructed in the use of thermal agents
• Receive walking aids, as needed
• Participate in tai chi programs
• Be instructed in the use of transcutaneous electrical stimulation*

No recommendations regarding the following:


• Participation in balance exercises, either alone or in combination with strengthening
exercises
• Wearing laterally wedged insoles
• Receiving manual therapy alone
• Wearing knee braces
• Using laterally directed patellar taping
• Be treated with traditional Chinese acupuncture

* These modalities are conditionally recommended only when the patient with knee
osteoarthritis (OA) has chronic moderate to severe pain and is a candidate for total
knee arthroplasty but either is unwilling to undergo the procedure, has comorbid
36

medical conditions, or is taking concomitant medications that lead to a relative or


absolute contraindication to surgery or a decision by the surgeon not to recommend
the procedure.

Carpal Tunnel Exam


Examine the wrist for swelling, warmth, tenderness, deformity, and discoloration.
Atrophy of the thenar eminence (the raised fleshy area on the palm of the hand near
the base of the thumb) may be evident with longstanding, untreated carpal tunnel
syndrome.
Tinel's sign: Tap over the median nerve at the wrist to reproduce symptoms.

Phalen's test: Flex wrist by having patient place dorsal surfaces of hands together in
front of her for 30 to 60 seconds to reproduce symptoms.

Carpal Tunnel Syndrome Diagnosis


Both Tinel's and Phalen's have fairly low sensitivity and specificity for diagnosing carpal
tunnel syndrome.
• Tinel's sign is about 36% sensitive and 75% specific for carpal tunnel syndrome.
• Phalen's is a little more sensitive at 57%, but less specific at 58% if the test is positive.
The diagnostic test of choice for carpal tunnel syndrome is an electrodiagnostic test (a
nerve conduction velocity study) which has a sensitivity of 48-84% and specificity of
95-99%.

One review found that the three most helpful findings in predicting the
electrodiagnosis of Carpal Tunnel Syndrome are:
• Hand symptom diagrams (patient indicates symptoms in at least 2 of digits 1,
(thumb) 2, and 3 ("classic" pattern), or with palmar symptoms as long as not
confined only to ulnar aspect of palm ("probable" pattern))
• Hypalgesia (decreased sensitivity to pain)
• Weak thumb abduction strength testing

Chronic Pain Medications


Opioids
A meta-analysis showed that strong opioids (oxycodone and morphine) were more
effective than naproxen or nortriptyline for treating chronic noncancer pain, but not the
weaker opioids (propoxyphene, codeine).
• The most common side effect of long-acting opioids is constipation. This side
effect can usually be treated with a bowel regimen that may include laxatives,
stool softeners, exercise, and a high water and fiber diet.
• Short-acting opioids act on several different receptors, including mu receptors in the
37

central nervous system. Their side effects are similar to the long-acting opioids,
and include euphoria, bradycardia, sedation, physical dependence, nausea,
vomiting, and respiratory depression.
• Short-acting opioids actually carry more risk of tolerance than long-acting ones (C)
because of their short half-life of three to four hours. Patients need to use them
more frequently to control their pain adequately. Short-acting opioids tend to be
helpful for flares of acute pain, but if daily use is needed, long-acting opioids
should be considered.

Tricyclic antidepressants
• Tricyclic antidepressants have anticholinergic side effects, including dry mouth,
constipation, urinary retention, blurred vision and paralytic ileus.
• They also have many gastrointestinal side effects, are sedating, and can have
neurologic side effects like ataxia, tremors, paresthesias, and mental
clouding.
• They are relatively contraindicated in patients with severe cardiovascular disease or
conduction problems because they can contribute to tachycardia,
arrhythmias, hyper- or hypotension, heart block, and myocardial infarctions.

Anticonvulsants
Anticonvulsants have been shown to be helpful for pain related to trigeminal neuralgia,
but evidence is lacking for other chronic pain syndromes. Lamotrigine (Lamictal) was
shown to be ineffective for treating chronic neuropathic pain in a meta-analysis.
• Some anticonvulsants require blood level monitoring and have severe side effects like
megaloblastic anemia.
• Carbamazepine (Tegretol) can interfere with other medications because it is a
cytochrome P-450 inducer, including decreasing the effectiveness of hormonal
contraception.
• Several anticonvulsants are also known teratogens.
Patients need to be educated about expectations for their pain control, and attainable
goals should be set.

Side Effects of NonSteroidal Anti-Inflammatory Drugs (NSAIDs)


• gastrointestinal upset
• decreasing the effectiveness of hypertension medications
• increasing the effect of sulfonylureas

Patients who use NSAIDs chronically, taking 5,000 or more pills, are at an increased risk
of developing end-stage renal disease. Elderly patients are at an increased risk of
developing gastric ulcers when using NSAIDs chronically. NSAIDs, aspirin, and
38

acetaminophen can all cause hepatotoxicity, and contribute to coagulopathy.


Recommended Chronic Pain Control When Initial Conservative Treatment
Tramadol, a centrally acting analgesic with effects on the μ-opioid receptor that also
stimulates release of serotonin and inhibits reuptake of norepinephrine, may be a
good choice, given its effectiveness in alleviating moderate to severe pain and its lower
abuse potential than other more potent opiod agonists, though it still carries some risk.
A long-acting opioid might also be a good option. Often a short-acting opioid is given
first to see how much is needed to control pain adequately over a 24-hour period, and
then is converted to a long-acting alternative. If the long-acting opioid alone is not
sufficient, then use either acetaminophen or a short-acting opioid for breakthrough
pain. The goal should always be to use the smallest sufficient dose for the shortest
period of time to achieve adequate pain control.
Tricyclic antidepressants can be effective for chronic pain treatment and may be
considered.

Recommended Preventive Screening for a 74-Year-Old Woman


Recommended Screening
Colorectal cancer screening by fecal occult blood test (FOBT), flexible sigmoidoscopy,
or colonoscopy is recommended for patients ages 50 to 75 years old. The US Preventive
Services Task Force (USPSTF) recommends against screening for colorectal cancer
after the age of 75 years.
The USPSTF recommends screening adults for depression in clinical practices that
have systems in place to assure accurate diagnosis, effective treatment, and followup.
Patients 18 years old and older should be screened for elevated blood pressure.
The USPSTF recommends biennial screening mammography for women ages 50 to
74. They state that evidence is insufficient to make any recommendation for screening
over the age of 75.

Screening Not Recommended


A one-time ultrasound to screen for an abdominal aortic aneurysm is recommended in
men 65-75 years old who have any history of smoking, but the USPSTF recommends
against routine screening for AAA in women.
Evidence is insufficient to make recommendations about the routine use of
interventions to prevent low back pain in adults in the primary care setting.
The USPSTF recommends against screening for carotid artery stenosis in
asymptomatic patients.
The USPSTF strongly recommends screening women over 45 years old for lipid
disorders if they are at increased risk of coronary heart disease. The USPSTF makes no
recommendation about screening women over the age of 20 who do not have an
increased risk of coronary heart disease.
39

There is not sufficient evidence to recommend for or against routine screening for
thyroid disease.
Based on USPSTF recommendations, women over the age of 65 should not be
screened for cervical cancer if they have had adequate recent screening with normal
Pap smears, and are not otherwise at high risk for developing the disease.

Annual Visit Preventive Medicine


1. Screening tests

2. Update adult immunizations
• Tetanus - Sustitute one-time dose of Tdap for Td booster, then boost with Td every
10 years
• Pneumococcal Polysaccharide - If > 65 years, one dose*
• Flu - One dose annually
• Zoster - If > 60 years, one dose
*The newest recommendations for adults aged 65 years or older who received
PPSV23 previously say to administer PCV13 at least 1 year after the dose of
PPSV23.

Case 13: 40M with a persistent cough (asthma, allergic rhinitis)

Causes of Chronic Cough


Common causes of persistent cough include:
• upper airway cough syndrome (UACS - previously called postnasal drip)
• vocal cord dysfunction
• asthma
• gastroesophageal reflux disease (GERD)
• cough due to medications such as angiotensin-converting enzyme inhibitors (ACE-
inhibitors)
• tobacco-related cough
• post-infectious cough
• chronic obstructive pulmonary disease (especially the chronic bronchitis type)
• non-asthmatic eosinophilic bronchitis.

Serious, less common causes of persistent cough include:


• pulmonary conditions such as bronchogenic carcinoma of the lung, sarcoidosis and
tuberculosis.
• cardiac conditions such as congestive heart failure.

Causes of Wheezing
Asthma is the most common cause of persistent cough and wheezing.
40

Other causes of wheezing to consider include:


• chronic obstructive pulmonary disease
• congestive heart failure
• foreign body aspiration
• persistent bronchitis
• upper airway cough syndrome
• vocal cord dysfunction
• pulmonary embolism

The following answers are incorrect:


• Pneumonia (E) may present with rales.
• Epiglottitis (H) may present with stridor.
• Obstructive sleep apnea (J) may present with daytime sleepiness, nocturnal snoring
and irregular breathing when asleep.

Acute sinusitis:
Symptoms include: Fever, headaches, facial pain, toothache, failure to respond to
decongestants, initial improvement after a viral URI and then a recurrence of
worsening symptoms
• In acute sinusitis, the nasal discharge is opaque and mucopurulent, not clear (clear
drainage may be associated with allergies).
• Many viral upper respiratory infections will gradually worsen over the five days. To
diagnose a patient with acute bacterial sinusitis, they should have symptoms
for a minimum of seven to ten days following a viral URI.
• Nasal congestion or obstruction persisting for > 12 weeks would be associated with
chronic sinusitis, not acute sinusitis.

According to the American Academy of Otolaryngology - Head and Neck Surgery


Foundation guidelines (2007) on sinusitis, making the distinction between a lingering
viral upper respiratory infection that affects the nose and sinuses (viral
rhinosinusitis) or an early acute bacterial sinusitis can be difficult.
It is more likely to be a viral rhinosinusitis if the duration of symptoms is less than
ten days and they are not worsening. In this case, you can continue to observe the
patient and reassure him that antibiotics are not necessary at this time.

Chronic sinusitis:
According to the AAO-HNS updated guidelines (2015), patients with chronic sinusitis
have similar symptoms to patients with acute sinusitis, but they last at least 12
41

weeks. They must have two of the following symptoms:


• nasal obstruction or congestion
• mucopurulent drainage (anterior, posterior or both)
• facial pain, pressure or fullness
• decreased sense of smell

They must also have signs of inflammation on physical examination or radiological


studies that will be discussed later.

However, according to the AAAAI/ACAAI Practice Parameter Update (2014), some


patients with chronic sinusitis may have "subtle" symptoms such as only a mild increase
in nasal congestion.

*21% of adults who have asthma have aspirin-induced asthma and should avoid
NSAIDs.

Differential Upper Respiratory Symptoms With New Onset Wheezing


Most Likely Diagnoses
Asthma with uncontrolled allergic rhinitis and possible chronic sinusitis is the most
likely diagnosis as the patient presents with upper respiratory symptoms and a new
onset of wheezing.
Other possible causes of his symptoms and signs include chronic obstructive
pulmonary disease, nonasthmatic eosinophilic bronchitis, and vocal cord
dysfunction since all of these conditions can cause cough and/or wheezing.
Another possible cause is gastroesophageal reflux. Asymptomatic gastroesophageal
reflux (confirmed by esophageal pH studies) can occur and can be associated with
asthma.

Less Likely Diagnoses


Congestive heart failure and pulmonary embolism are very unlikely in the setting of lack
of other symptoms and signs that one would expect to see with them on history and
physical examination. Mr. Dennison denies shortness of breath, a major symptom of
both conditions. His oxygen saturation is normal, making pulmonary embolism
unlikely. While patients with congestive heart failure may have cough and/or wheezing,
Mr. Dennison has no signs of congestive heart failure such as crackles in the lungs or
bilateral ankle edema.

Studies to Evaluate Upper Respiratory Symptoms With New Onset Wheeze


Spirometry: findings will help us rule in or rule out several of the diagnoses on the
differential.
42

Tests not indicated at this time


Chest x-ray: A chest x-ray is useful in the evaluation of a patient with a persistent
cough to rule in or rule out a suspected infection such as pneumonia. On a chest x-ray,
you may find hyperinflated lungs on a patient with asthma, but this is not specific (since
COPD can also cause this).
CT scan of chest: A CT scan of the chest would be helpful if there were an abnormality
on the chest x-ray, such as a lung mass or if there was a concern about a pulmonary
embolus, but is unlikely to be helpful in a patient presenting with wheezing.
Methacholine challenge: A methacholine challenge test may be considered if the
spirometry findings are normal. The National Asthma Education and Prevention
Program Expert Panel Report 3 (2007) suggests that this test should be carried out by a
"trained individual" and therefore the patient most likely will be referred out for this
test.
Ventilation-perfusion scan: While there can be a ventilation-perfusion mismatch in
patients with asthma, it is unlikely that a ventilation-perfusion scan is helpful in the
initial management of asthma. A ventilation-perfusion scan is more useful in
diagnosing pulmonary embolism.

Suspected Asthma Management


The American College of Chest Physicians (ACCP) recommends offering standard
asthma medication including an inhaled bronchodilator and an inhaled
corticosteroid.

Allergic Rhinitis Management


• Oral antihistamine
• Inhaled nasal corticosteroid

For longstanding allergic rhinitis


It is reasonable to continue oral antihistamine (cetirizine) (C) and inhaled nasal
corticosteroid (fluticasone) (E) while you continue to evaluate his asthma. You
encourage him to take his medications more regularly so that you can gauge their
effect on his next visit.
For cough due to suspected asthma
Since the patient is not in distress and will be seen again soon, it is reasonable to offer
him only an inhaled bronchodilator (D) at this point.

While cough suppressants such as dextromethorphan (A) or codeine (B) can be offered
to a patient with an acute cough, it is better to give an albuterol inhaler to Mr.
Dennison since he has a chronic cough that is likely due to asthma.
43

FEV1 value = Volume of air exhaled during the first second of forced exhalation
following maximal inhalation.
FVC value = Maximal volume of air forcibly exhaled from the point of maximal
inhalation.

Differential of Obstructive Lung Disease With Reversible Findings


Most Likely Diagnosis
Based on spirometry findings of obstructive lung disease and improvement following
treatment with inhaled corticosteroids, asthma is the most likely diagnosis.
The National Asthma Education and Prevention Program Expert Panel Report 3
(2007) states that an improvement in the FEV1 value by >12% or an increase in the
percent predicted FEV1 value by 10% after a bronchodilator is given is diagnostic for
asthma.
Reversible obstructive findings on spirometry is the distinctive diagnostic abnormality
in patients with asthma, especially early in the course. Patients with chronic, severe
asthma may have less or no reversibility of their obstructive findings, very similar to
patients with chronic obstructive pulmonary disease.

Less Likely Diagnoses


• Patients with nonasthmatic eosinophilic bronchitis will respond to inhaled
corticosteroids like patients with asthma, but they will have a normal spirometry
and normal chest x-ray. The diagnostic finding for this condition is sputum
eosinophilia on induced sputum or bronchial wash obtained at bronchoscopy.
• Patients with vocal cord dysfunction may have flattening of the inspiratory loop on
spirometry, but do not typically have reversible obstructive findings on
spirometry like patients with asthma. The diagnostic finding of this condition is
visualizing abnormal vocal cord movement during an episode of wheezing.
• Symptoms of chronic obstructive pulmonary disease, like those of asthma, should
improve following treatment with bronchodilators and inhaled
corticosteroids. Obstructive findings are seen on spirometry with this
condition, however, should not be reversible.
• Patients with gastroesophageal reflux disease typically present with either heartburn
symptoms or findings of esophagitis on upper endoscopy. Even if asymptomatic,
reflux can trigger bronchoconstriction and serve as an exacerbating factor for
patients with asthma. If a patient with asthma fails to improve with standard
treatment, it is reasonable to consider whether gastroesophageal reflux is
present.

Asthma severity chart


44

When a patient experiences difficulty with asthma control, the physician must consider
and address comorbid conditions known to effect asthma control. These comorbid
conditions include: gastroesophageal reflux (GERD), chronic sinusitis / uncontrolled
allergic rhinitis, stress / depression, obstructive sleep apnea, and being overweight
or obese. Successful treatment of these conditions often results in improved control of
the patient’s asthma symptoms.

• High dose inhaled corticosteroids are reserved for severe asthma as the risk of
adverse effects increase with dose. Inhaled corticosteroids are well-tolerated
and safe at the recommended doses. To reduce the potential adverse effects of
inhaled corticosteroids: spacers are recommended to reduce local side effects,
patients are advised to rinse their mouths and spit after inhalation, and consider
adding a long-acting beta agonist to a low- or medium-dose of inhaled
corticosteroid rather than using a higher dose of corticosteroid.
• Oral corticosteroids suppress, control, and reverse airway inflammation. However,
side effects with chronic administration include among other things:
osteoporosis, adrenal suppression, growth suppression, dermal thinning,
hypertension, Cushing's syndrome, cataracts, increased emotional lability,
psychosis, peptic ulcer disease, atherosclerosis, aseptic necrosis of the bone,
diabetes mellitus, and myopathy. Every effort, then, is given to minimizing
systemic corticosteroid use and maximizing other modes of therapy. When oral
corticosteroids are resorted to (for quick relief of symptoms in a moderate or
severe asthma exacerbation), they are given for a short duration, and side
effects are monitored. Multiple courses of oral systemic corticosteroids (more
than three courses annually) should prompt re-evaluation of asthma
management for the patient.
• Leukotriene receptor antagonists may be used in conjunction with low-dose inhaled
corticosteroids, but they are expensive. A Cochrane review also revealed as
additions to patients already on inhaled corticosteroids, a long-acting beta2
agonist inhaler improves symptoms and lung function, while preventing
exacerbations, more effectively than leukotriene receptor antagonists.
• Theophylline may also be used in conjunction with low-dose inhaled corticosteroids,
but is not used that often due to the difficulty in titrating the theophylline dose
to the correct level. A meta-analysis demonstrated that salmeterol (a long-
acting beta2 agonist) inhaler led to improved lung function and more symptom-
free days and nights compared to theophylline.

Pharmaceutical Therapy for Allergic Rhinitis


Nasal corticosteroids are the most effective medications for patients with allergic
45

rhinitis. In one systematic review, nasal corticosteroids improved symptoms of nasal


blockage, nasal discharge, sneezing, nasal itch and post-nasal drip compared to
antihistamines.
Similarly, a Cochrane review demonstrated that allergen immunotherapy (allergy
shots) assists in controlling symptoms and decreasing medication use in patients with
seasonal allergic rhinitis. A third Cochrane review demonstrated that allergen
immunotherapy is effective in improving asthma symptoms and decrease asthma
medication usage.

Influenza Vaccination
Annual vaccination against influenza is recommended for all persons aged 6 months
or older.

Tetanus, Diphtheria, and Acellular Pertussis (Td/Tdap) Vaccination In Adults


• Persons aged 11 years or older who have not received Tdap vaccine or for whom
vaccine status is unknown should receive a dose of Tdap followed by tetanus and
diphtheria toxoids (Td) booster doses every 10 years thereafter. Tdap can be
administered regardless of interval since the most recent tetanus or diphtheria-
toxoid containing vaccine.
• Adults with an unknown or incomplete history of completing a 3-dose primary
vaccination series with Td-containing vaccines should begin or complete a
primary vaccination series including a Tdap dose.
• For unvaccinated adults, administer the first 2 doses at least 4 weeks apart and the
third dose 6 to 12 months after the second.
• For incompletely vaccinated (i.e., less than 3 doses) adults, administer remaining
doses.

Pneumococcal 23-Valent Pneumococcal Polysaccharide Vaccine [PPSV23]


Vaccination In Adults with Chronic Disease
Adults aged 19 through 64 years with chronic heart disease (including congestive heart
failure and cardiomyopathies, excluding hypertension), chronic lung disease (including
chronic obstructive lung disease, emphysema, and asthma), chronic liver disease
(including cirrhosis), alcoholism, or diabetes mellitus: Administer PPSV23.

According to the AAO-HNS updated guidelines (2015), to make a diagnosis of


chronic sinusitis, the patient must have at least 12 weeks of at least two of the
following symptoms: nasal obstruction/congestion, mucopurulent drainage,
facial pain, pressure or fullness, or a decreased sense of smell. In addition,
inflammation must be demonstrated by one of the following: purulent mucus or
46

edema in the middle meatus or ethmoid region, polyps in the nasal cavity or
middle meatus area, or inflammation of the sinuses on radiographic imaging.
Therefore, being able to demonstrate any inflammation on either nasal
endoscopy or a CT of the sinuses will confirm the diagnosis of chronic
sinusitis.
Nasal endoscopy (Not performed in this case) is useful in demonstrating
inflammation of the nasal mucosa, nasal polyps or other masses, the presence of
nasal secretions and/or purulence, anatomic deformities.
A CT of the sinuses may show inflammation of the paranasal sinuses, masses
including sinonasal polyps, anatomic deformities. A CT scan of the sinus does
not necessarily correlate with the severity of the patient's symptoms, but is an
objective method to monitor chronic or recurrent sinus disease.
Multiple studies have linked chronic sinusitis to asthma. For example, one
recent paper found that 42% of patients with chronic sinusitis have asthma.
There also are studies reporting similar pathologic processes in chronic sinusitis
and asthma. Here are some references, which discuss this in further detail.
One recent study found that the use of a nasal steroid, mometasone, did not
improve symptoms of asthma control, but this study only evaluated the use of
this medication for 24 weeks. A study of longer duration is needed to evaluate if
there are more favorable outcomes of asthma if chronic sinusitis symptoms are
treated long-term. In the meantime, the AAAAI/ACAAI Practice Parameter
Update (2014) recommends that treatment of rhinosinusitis symptoms be
treated "vigorously" due to their beneficial effect on asthma symptoms.
(Strength of recommendation: Class C)

Case 15: 42M with RUQ pain (biliary colic, alcoholism)

Abdominal Exam - Appendicitis


Psoas sign - Passive extension of patient's thigh as s/he lies on his/her side with knees
extended, or asking the patient to actively flex his/her thigh and hip causes abdominal
pain, often indicative of appendicitis.

Obturator Sign - Examiner has patient supine with right hip flexed to 90 degrees- takes
patient's right ankle in his right hand as he uses his left hand to externally/internally
rotate patients hip by moving the knee back and forth. Elicitation of pain in the
abdomen implies acute appendicitis.

CAGE & AUDIT Screening for Alcohol Abuse/Dependence


47

The CAGE questionnaire is a classic series of screening questions for the likelihood of
alcohol abuse and/or dependence.

Positive answers to two or more of the CAGE questions are sufficient to identify
individuals who require more intensive evaluation. Also, a positive answer to the
question, "Have you ever had a drinking problem?" plus evidence of alcohol
consumption in the previous 24 hours provides greater than 90% sensitivity and
specificity as a screening tool for identifying alcoholism.

The CAGE questionnaire has consistently proved to be a useful instrument for detecting
alcohol abuse and alcohol dependence.

Modified CAGE:
Have you ever felt:
1. The need to Cut down on drinking?
2. Annoyed with criticisms about your drinking?
3. Guilt about your drinking?
4. The need to drink an Eye opener in the morning?

Furthermore, the American Society of Addiction Medicine has developed standards for
a positive screen based on the number of drinks ingested per week. Consumption of
more than 14 drinks per week or more than 4 drinks per occasion for men, and more
than 7 drinks per week or more than 3 drinks per occasion for women is considered a
positive screen.
The AUDIT-C is another example of a screening questionnaire. It is a three-item alcohol
screen that can help identify persons who are hazardous drinkers or have active alcohol
use disorders (including alcohol abuse or dependence).

Differential of Right Upper Quadrant Abdominal Pain


Biliary colic typically causes right upper quadrant pain, epigastric pain or chest pain
that is constant (the term "colic" is a misnomer), typically lasts 4-6 hours or less, and
often radiates to the back (classically under the right shoulder blade). It is often
accompanied by nausea or vomiting and often follows a heavy, fatty meal. These
symptoms are a result of a stimulated gallbladder (e.g., from a fatty meal) contracting
when a gallstone obstructs the outlet of the cystic duct. The hallmark of biliary colic is
that the stone is mobile and eventually moves away from the outlet allowing
resumption of normal gallbladder function and resolution of symptoms.

RUQ pain from cholecystitis also causes right upper quadrant pain with associated
nausea and vomiting and also classically occurs following a large, fatty meal. The
48

pathophysiology of cholecystitis is similar to biliary colic but is caused by a stone that is


not dislodged from the cystic duct outlet. In contrast to biliary colic, the symptoms of
cholecystitis typically persist, are more severe, and are often associated with fever.
An elevated white blood cell count is often present from inflammation of the distended
gallbladder wall. It should be noted that these inflammatory changes found with
cholecystitis can be acute or chronic. Additionally, the condition of acalculous
cholecystitis is recognized, particularly in the elderly and the very-ill ICU patient.
Duodenal ulcer typically causes epigastric pain (possibly right or left upper quadrant
pain) that is relieved rather than worsened by food and is relieved by antacids. While
indigestion and/or nausea are common, vomiting and radiation to the back can occur
but are uncommon. However, there can be significant variation in symptoms.
The clinical manifestations of hepatitis vary somewhat depending on etiology, but
most types do not have acute onset, unlike the diseases of the gallbladder tract. While
RUQ pain, nausea, and vomiting are frequently encountered, there is often associated
malaise, anorexia, itching, and icterus/jaundice. Hepatomegaly is often present.

Nausea, vomiting and epigastric pain are hallmarks of acute pancreatitis. Typically,
however, there is abdominal tenderness on exam and there is unlikely to be resolution
of symptoms without prolonged bowel rest, and jaundice may be seen if there is
obstruction of the common bile duct. Distinguishing acute pancreatitis from biliary colic
(and any other upper abdominal disease) can be challenging, particularly because the
two most common etiologies of acute pancreatitis are alcoholic pancreatitis and
gallstone pancreatitis. Gallstone pancreatitis may be preceded by an episode of biliary
colic. With the onset of acute pancreatic inflammation as the pancreatic duct is
obstructed, the pain worsens rapidly and radiates to the back. Some classic, though
rare, physical exam signs seen in acute pancreatitis include:
• Grey-Turner's sign: ecchymotic discoloration in the flank
• Cullen's sign: ecchymotic discoloration in the periumbilical region

Finally, the presentation of acute pancreatitis may include shock and/or coma.

The following diagnoses are less likely:


Pneumonia or pleurisy (C) is less likely given the absence of respiratory symptoms
such as cough or pleuritic pain and the absence of signs such as crackles on the
pulmonary auscultation portion of the exam. Additionally, the absence of fever and
night sweats makes pulmonary TB rare as well.
Myocardial infarction (D) deserves serious consideration given Mr. Keenan's family and
social history. However, there is no chest pain, and the description of the epigastric and
RUQ pain is not consistent with classic anginal pain. Typical angina pain would be
defined as substernal chest discomfort characterized by all of the following
49

characteristics: a typical quality and duration, provocation by exertion or emotional


stress, and relief by rest or nitroglycerin.
Renal pain or colic (E) is less likely because of the steady, non-colicky nature of the
pain Mr. Keenan describes, but again, deserves consideration.


Pyelonephritis (G) is unlikely based on the absence of a fever, the lack of costo-
vertebral angle tenderness, and the absence of urinary symptoms.


Herpes zoster (J) can present without visible skin lesions in the earliest stages.
However, the absence of skin lesions makes this diagnosis unlikely and there is often a
prodromal phase associated with worsening pain and hyperaesthesia of the skin
progressing to visible skin lesions of that particular dermatome on examination.


Appendicitis (K) is impossible due to Mr. Keenan's history of appendectomy.
Additionally, it is important to point out that Mr. Keenan does not have any classic
peritoneal signs, which would include abdominal rigidity visible during the observation,
percussion and gentle palpation portion of the exam.

Studies to Evaluate Right Upper Quadrant Abdominal Pain


CBC (complete blood count) is indicated to assess for leukocytosis that would suggest
infection and to assess for anemia that might suggest internal bleeding.
Electrolytes are indicated to assess for electrolyte alterations due to vomiting.
LFT (liver function testing) is indicated to assess for acute or chronic liver damage
(ALT and AST) and to assess for biliary tract involvement (alkaline phosphatase and
total bilirubin).
UA (urinalysis) is useful in assessing for urinary red blood cells that might suggest renal
colic.
Amylase/lipase are indicated to assess for pancreatitis.

Inappropriate diagnostic tests at this time


D-dimer (C) has no role in the absence of symptoms or signs of deep venous thrombosis
or pulmonary embolism, such as unilateral lower extremity edema, tachypnea,
tachycardia, and pleuritic chest pain.
CXR (chest x-ray) (E) could be considered to rule out a RLL pneumonia such as
mycoplasma (aka "walking pneumonia"). In the absence of a cough or pulmonary
findings, many would not include it at this juncture.
ESR (erythrocyte sedimentation rate) (F) is not indicated as a chronic inflammatory
process is not being considered.
Abdominal plain films (G) have little role in Mr. Keenan's specific situation. His
symptoms suggest neither an acute intestinal obstruction nor free air from a perforated
hollow viscus, the primary two conditions where plain abdominal imaging are useful. If
the urinalysis showed hematuria, that would increase the likelihood (currently low) of
50

renal colic, and a simple KUB (kidney, ureter, bladder) film would indeed be helpful in
ruling in or ruling out a kidney or ureteral stone.
EKG (H) and Troponin I (I) could be done given Mr. Keenan's family and social history
and the frequent atypical presentations (i.e., without chest pain) of myocardial
infarction and coronary artery disease in general. However, neither of these tests would
rule out coronary artery disease and most physicians would defer these tests at this
time.
BNP (Brain natriuetic peptide) (J) is primarily useful in distinguishing chronic obstructive
pulmonary disease from congestive heart failure, especially in the evaluation of
shortness of breath, and would not be useful here.
Management of Biliary Colic
Surgical consultation for cholecystectomy.

If the patient instead had acute cholecystitis, rather than an episode of biliary colic,
urgent cholecystectomy may be required. Typically, a less urgent surgical approach is
undertaken with a transient episode of biliary colic. However, there is emerging
evidence that cholecystectomy within the first 24 hours of the diagnosis of an
episode of biliary colic has merit.

Expectant management or "watchful waiting" would not be appropriate as natural


history studies document a 70% risk of progression over 2 years to complications such
as cholangitis, pancreatitis, cholecystitis, choledocholithiasis, gallstone ileus and Mirizzi
syndrome (gallstone compression of the hepatic duct).

A three-month trial of ursodiol (Actigall) - an agent that is sometimes effective in


dissolving gallstones and preventing future gallstones - would be equivalent to watchful
waiting in many clinical circumstances. If the patient is having more atypical symptoms
with visible stones, which may or may not account for the symptoms, then such a
medical trial might be warranted. If symptoms resolve, then they may have been from
the gallstones and subsequent therapy can be planned. If symptoms do not resolve,
then plans can likewise be made.

Further imaging is probably not warranted. If the patient has typical symptoms of
biliary colic but no visible stones on the gallbladder ultrasound, a HIDA scan might be
obtained to look for gallbladder dysfunction and reproducible pain.
If there was jaundice and/or gallstone pancreatitis suggestive of a common duct
stone (choledocholelithiasis), an ERCP might be warranted.

Another role for ERCP would be in the postoperative patient who did not have an
intraoperative cholangiogram (assessing the common duct at the time of surgery) and
51

who presents with a repeat episode of biliary colic and/or jaundice and/or pancreatitis.
MRCP is a similar diagnostic modality that uses magnetic resonance. However, unlike
ERCP - where treatment can take place at the time of diagnosis… MRCP is a diagnostic
modality only.

Classifying Alcohol Use


"Risky/hazardous drinking" means the patient's alcohol consumption exceeds the
National Institute on Alcohol Abuse and Alcoholism per occasion threshold for men of 4
drinks per occasion.
"Problem drinking" entails significant physical, social, or psychological harm from
drinking.
"Alcohol abuse" requires a maladaptive pattern of use with one or more of the
following:
• Failure to fulfill work, school, or social obligations
• Recurrent substance use in physically hazardous situations
• Recurrent legal problems related to substance use
• Continued use despite alcohol-related social or interpersonal problems

"Alcohol dependence" requires three or more of the following:


• Tolerance
• Withdrawal
• Substance taken in larger quantity than intended
• Persistent desire to cut down or control use
• Significant time spent obtaining, using, or recovering from alcohol use
• Social, occupational, or recreational tasks are sacrificed
• Use continues despite physical and psychological problems

As in many areas of medicine, the differentiation among these categories of alcohol use
is somewhat overlapping. Most agree that any of the patterns are an indication for
physician counseling and intervention.

Intervention and Treatment of Risky / Hazardous Drinking Behavior


• Brief intervention by the family physician consisting of a 10-15 minute session with
advice and goal-setting. Usually this is followed by return visit or phone call. This
is the most likely option to choose for a patient in a rural area.
• Referral for MET consisting of 4 sessions over 12 weeks utilizing techniques of
motivational interviewing. This intervention requires more of a commitment
from the patient to attend sessions. This modality may not be available in a rural
area
• Referral for CBT, a structured form of psychotherapy that works to improve the
52

patient's awareness of his behavior and to develop new, more adaptive


behaviors. Also requires patient commitment and may not be readily accessible
in a rural area.
• Voluntary participation in AA. This program is available in almost all regions and
utilizes group support and a Twelve-Step process emphasizing total alcohol
abstinence.

Incorrect answers include:


• Inpatient treatment program (D). This patient's pattern of alcohol use and absence of
withdrawal symptoms suggests he does not need an intensive inpatient
program.
Intensive outpatient treatment program (E). An intensive program is typically used for
patients with more extensive psycho-social co-morbidities.

Likelihood of Maintaining Sobriety


The statistics are limited, but we do know that higher rates of relapse occur in
individuals who are: men, younger age, have fewer social supports, who drank
more prior to treatment and when applicable, have poor compliance with drug
therapy.

Case 18: 24F with headaches (Tension and migraine)

Defining Characteristics of Primary Headaches


Migraine Tension type Cluster
Severity of Moderate to Mild to Severe.
pain severe. moderate.
Associated Often occur with May occur with Associated with rhinorrhea, lacrimation,
symptoms nausea and photophobia or facial sweating, miosis, eyelid edema,
vomiting, hyperacusis. conjunctival injection, and ptosis.
photophobia, or
hyperacusis. May
occur with aura.
Quality of Pulsating and can Pressing, Severe unilateral orbital, periorbital,
pain be unilateral. tightening, and supraorbital, or temporal pain.
bilateral.
Aggravating Worsened with Typically not
factors physical activity. worsened with
53

physical
activity.
Duration of Last from 4-72 Last from 30 Last 15-180 minutes.
symptoms hours. minutes to 7
days.
Number of 5 episodes needed 10 episodes 5 episodes needed for diagnosis.
episodes for diagnosis. needed for
diagnosis.

Secondary Headaches

Etiology of secondary headache Findings


Meningitis: Headache with fever, mental status changes, or stiff neck.
Intracranial hemorrhage: Sudden onset of headache, severe headache, recent trauma,
elevated blood pressure.
Brain tumor: Cognitive impairment, weight loss or other systemic symptoms,
abnormal neurologic examination.
Traumatic brain injury (concussion): Head injury with subsequent confusion and
amnesia. Loss of consciousness sometimes occurs. Subsequent headache, dizziness,
and nausea and vomiting. Over hours and days: mood and cognitive disturbances,
sensitivity to light and noise, and sleep disturbances.

Common Etiologies of Secondary Headaches


1. Headache due to depression or anxiety

Features
 Similar to tension type headache:


• Bilateral, pressing, and/or tight
• Last from 30 minutes to 7 days

Some experts feel that depression or anxiety can trigger tension type headaches. In
those cases tension-type headaches are considered secondary, not primary headaches.

2. Medication overuse headache (also called analgesic rebound headache)


Chronic use of any analgesic can cause this type of headache.

Features
• Mild to moderate in severity
• Diffuse, bilateral headaches that occur almost daily and are often present on first
waking up in the morning.
54

• Often aggravated by mild physical or mental exertion.


• Can be associated with restlessness, nausea, forgetfulness, and depression.
• Headaches may improve slightly with analgesics but worsen when the medication
wears off. Tolerance develops to abortive medications and there is decreased
responsiveness to preventive medications.
• Medication overuse headache can occur at varying doses for different types of
medication; it may occur with as low as an average of 18 doses of triptans per
month, but may require as high as an average of 114 doses of analgesics per
month.

Diagnostic criteria
• More than 15 headaches per month.
• Regular overuse of an analgesic for more than three months.
• Development or worsening of a headache during medication overuse.
• Headache resolves or reverts to its previous pattern within 2 months after
discontinuation of overused medication.

Treatment
Stopping the overused medication.

Indications for Brain Imaging in the Evaluation of Headache


Don't do imaging for uncomplicated headache. Imaging headache patients absent
specific risk factors for structural disease is not likely to change management or
improve outcome. Those patients with a significant likelihood of structural disease
requiring immediate attention are detected by clinical screens that have been validated
in many settings. Many studies and clinical practice guidelines concur. Also, incidental
findings lead to additional medical procedures and expense that do not improve patient
well-being. For more information, see the "Choosing Wisely" campaign of the American
Board of Internal Medicine Foundation.

The American Academy of Neurology and the U.S. Headache Consortium guidelines
recommend neuroimaging only if:
• The patient has migraine with atypical headache patterns or unexplained
abnormalities on neurological examination
• The patient is at higher risk of a significant abnormality
• The results of the study would alter the management of the headache

Symptoms that increase the odds of positive neuroimaging results include:


• Rapidly increasing frequency of headache
• Abrupt onset of severe headache
55

• Marked change in headache pattern


• A history of poor coordination, focal neurologic signs or symptoms, and a headache
that awakens the patient from sleep.
• A headache that is worsened with use of Valsalva's maneuver
• Persistent headache following head trauma
• New onset of headache in a person age 35 or over
• History of cancer or HIV

Screening for Anxiety and Depression


The two questions you asked are a screening tool for anxiety in the primary care setting
known as the GAD-2. There is a similar screening tool for depression known as the PHQ-2.

GAD-2
Over the last two weeks, how Not at all Several Nearly half Nearly every day
often have you been bothered by days the days
the following problems?
1. Feeling nervous, anxious or on 0 1 2 3
edge?
2. Not being able to stop or control 0 1 2 3
worrying?
(For office scoring, total score T__ _____ + ____ + ___ )
=
A positive screening test is a score >3 points.

PHQ-2
Over the last two weeks, how Not at all Several More than Nearly every day
often have you been bothered by days one-half the
the following problems? days
Little interest or pleasure in doing 0 1 2 3
things
Feeling down, depressed, or 0 1 2 3
hopeless
A negative response to both questions is considered a negative result for depression.

A positive response to either question in the PHQ-2 or the GAD-2 is highly sensitive for either
depression or anxiety, respectively. However neither test is very specific.

If a patient has a positive response to one of the questions, a more comprehensive screening
tool, the PHQ-9 or the GAD-7, must be administered.
These longer questionnaires are more specific in identifying depression or anxiety.

Important Physical Exam Findings with Headache


56

Signs of increased intracranial pressure -


• Papilledema
• Altered mental status

Other important findings to look for -


• Signs of meningeal irritation such as Kernig's sign or Brudzinski's sign
Focal neurologic deficits such as unilateral loss of sensation, unilateral weakness, or
unilateral hyperreflexia.

How to Perform a Neurological Exam


Test cranial nerves II through XII:
Cranial Test
Nerves
II and Pupils are equal, round, and reactive to light.
III
II Test visual fields with confrontation.
III, IV, Extraocular eye movements are intact. Convergence intact.
and VI
V Ask the patient to close her eyes and then ask if the two stimuli feel the same
when you lightly touch her right, then left forehead; right, then left cheek;
right, then left chin.
VII Observe for facial asymmetry while the patient is talking or performing the
following maneuvers:
• Raise her eyebrows.
• Frown.
• Close both eyes tightly while you try to open them.
• Show both upper and lower teeth.
• Smile.
• Puff out both cheeks.
VIII Rub your fingers near each ear.
XI Ask the patient to elevate her shoulders against resistance.
IX, X, Note if speech is clear and tongue and palate are midline.
and XII

Complete the neurologic exam:


• Light touch: Sensation to light touch on all extremities.
• Motor: Assess power and symmetry in all extremities. Look for rigidity or clonus. Test
flexor plantar response bilaterally.
• Reflexes: Deep tendon reflexes.
• Cerebellar exam: Look for finger-to-nose or heel-to-shin ataxia.
57

Assess gait by having the patient walk toward you while walking on her heels, then walk
away from you on her tiptoes. Then you have her walk in tandem, placing one foot
directly in front of the other as if walking on a tightrope.

Findings suggestive of migraine headaches:


• Severe headaches one to two times weekly
• Unilateral and throbbing in nature
• Associated nausea, photophobia and hyperacusis
• Family history of migraines
• Increased caffeine consumption and poor sleep are possible triggers
• Normal neurologic exam

Findings suggestive of tension type headaches:


• Mild to moderate bilateral headaches that last all day on most days
• Pain that radiates down neck with tender occiput
• Increased stressors and poor sleep in recent weeks are possible trigger
• Normal neurologic exam

Unlikely Rationale
diagnosis
Cluster Sarah's pain is not associated with autonomic features.
headache
Bacterial Unlikely due to chronic nature of her headache and absence of fever,
meningitis lack of changes in mental status or history of recent illness.
Additionally, she had no findings concerning for meningitis on her
physical exam.
Intracranial No history of recent trauma or a change in the pattern of her headaches.
hemorrhage Her neurologic exam was normal as well. Patients with ICH often describe
"worst headache of my life".
Brain tumor Age <50, lack of systemic symptoms, and the absence of focal neurologic
deficits. Brain tumor unlikely without change in headache pattern.
Brain tumor should be considered in a first headache in a patient >50 years
old.
Medication Not using analgesics frequently enough to fit this diagnosis. For simple
overuse analgesics such as ibuprofen, patients must report using the medication
headache at least fifteen times per month for three months. Sarah reported taking
ibuprofen twice weekly.
Headache Neither of these conditions is present.
due to
58

anxiety or
depression

Triggers for Tension & Migraine Headaches


Physical or environmental triggers:
• Intense or strenuous exercise
• Sleep disturbance
• Menses
• Ovulation
• Pregnancy (though for many women, headaches actually improve during pregnancy)
• Acute illness
• Fasting
• Bright or flickering lights
• Emotional stress

Medications or substances that act as triggers:


• Estrogen (birth control/hormone replacement)
• Tobacco, caffeine or alcohol
• Aspartame and phenylalanine (from diet soda)

Possible dietary triggers:


• Ripened cheeses
• Alcohol, especially beer and red wine
• Pickled or fermented foods
• Monosodium glutamate (MSG)
• Yeast-based products
• Chocolate
• Legumes and beans
• Onions
• Citrus fruits
• Bananas

Generic/Trade Contraindications Side Effects


Names
Migraine-Specific
triptans sumatriptan (Imitrex, ergotamine, MAOIs, dizziness, sleepiness,
Imigran), naratriptan history of hemiplegic nausea, fatigue,
(Amerge, Naramig), or basilar migraine, paresthesia, throat
rizatriptan (Maxalt), significant tightness/closure,
zolmitriptan (Zomig), cardiovascular, chest pressure
59

frovatriptan (Frova, cerebrovascular, or


Migard), almotriptan peripheral vascular
(Axert), eletriptan disease, severe
(Relpax) hypertension,
pregnancy, in
combination with
SSRI's may cause
serotonin syndrome
ergot alkaloids ergotamine triptans, many severe reactions possible;
(Ergostat), possibly serious drug MI, ventricular
ergotamine/caffeine interactions; heart tachyarrhythmias, stroke,
(Cafergot), disease or angina, hypertension, nausea,
dihydroergotamine hypertension, vomiting, diarrhea,
(DHE) peripheral vascular dry mouth, rash
disease, pregnancy,
renal insufficiency,
breastfeeding
Non-Specific
Treatments Trade Name
(Effective for any
pain disorder)
Generic Name

aspirin/butalbital/ca Fiorinal history of porphyria or anaphylaxis, toxic


ffeine peptic ulcers, epidermal necrolysis,
bleeding risk; caution Stevens-Johnson syndrome, my
in drug abuse; thrombocytopenia,
pregnancy GI bleed
acetaminophen/but Esgic, Fioricet, history of porphyria, dizziness, drowsiness,
albital/caffeine Phrenilin (lacks pregnancy; caution in dyspnea, nausea, vomiting,
caffeine) drug abuse abdominal pain;
agranulocytosis,
thrombocytopenia,
respiratory depression,
Stevens-Johnson syndrome
acetaminophen/dic Midrin (discontinued hepatorenal hypertension, dizziness,
hloralphenazone in the US) insufficiency, rash
diabetes,
hypertension,
glaucoma, heart
60

disease, MAOI use


acetaminophen/ Excedrin Pregnancy; sensitivity nausea; GI bleed;
aspirin/caffeine to aspirin hypertension

Note: Don't use opioid or butalbital treatment for migraine except as a last resort.
Opioid and butalbital treatment for migraine should be avoided because more
effective, migraine-specific treatments are available. Frequent use of opioid and
butalbital treatment can worsen headaches. Opioids should be reserved for those with
medical conditions precluding the use of migraine-specific treatments or for those who
fail these treatments.

Migraine prophylaxis
Patients who have migraines more frequently than twice weekly are at risk for
medication overuse headache. Migraine prophylaxis should be considered in these
patients if the lifestyle changes aren't effective.

Drugs used FDA Efficacy/co Contraindic Side effects


(daily dose Approved? st ations /
range) Cautions
Beta- First line: Yes Good- Asthma, Fatigue, bronchospasm, light
blockers Metoprolol excellent/c depressions insomnia, bradycardia,
(47.5-200 mg) heap evere depression, sexual
Propranolol COPD, DM dysfunction
(20-160mg) requiring
Timolol (10- insulin,
30mg) Raynaud's
disease
Second line:
Atenolol
Nadolol
Neurostab First line: Yes Good/expe Pregnancy/r Divalproex: birth defects,
ilizers Divalproex nsive isk of weight gain, alopecia,
sodium (500- pregnancy pancreatitis, ovarian
1500mg); Topi cysts
ramate (25- Gabapentin Topiramate: renal stones,
200mg category C weight loss
Gabapentin: dizziness,
Divalproex: somnolence
hepatic
61

disease
Tricyclic Second line: No (off- Excellent/c Cardiac Drowsiness, weight gain,
Antidepres Amitriptyline label) heap and conduction dry mouth
sants (10-150mg) also work defects,
for MAOI
fibromyalg
ia and
tension-
type
headache
Herbal Butterbur No Cheap hepatotoxi belching, headache,
(100-150mg) city, itchy eyes, GI issues,
allergic asthma, fatigue
reactions in
patients
with plant
allergies,
safety not
established
for long-
term use

When to Initiate Prevention of Migraines


The American Migraine Prevalence and Prevention Study outlines recommendations as
to when daily pharmacological treatment should be initiated:
• At least six headache days per month
• At least four headache days with at least some impairment
• At least three headache days with severe impairment or requiring bed rest.

Prevention should be considered:


• Four to five migraine days per month with normal functioning
• Two to three migraine days per month with some impairment
• Two migraine days with severe impairment.

DSM-V Substance Use Disorder


The DSM-V substance use disorder criteria combine the DSM-IV criteria for
dependence, addiction, and tolerance. There is now one term, "substance use
disorder," that encompasses a continuum of problems with substances from mild to
severe. Each specific substance use disorder is diagnosed in similar fashion, using a list
62

of 11 symptoms to determine the severity of illness.

For opioid use disorder, the 11 symptoms are:


• opioids taken in larger amounts than intended
• unsuccessful efforts to control use
• significant time spent in opioid-related activities
• craving
• use results in unmet obligations at work, school, or home
• continued use despite significant interpersonal problems related to use
• other activities neglected due to use
• use in physically hazardous situations
• continued use despite physical or psychological problems related to use
• tolerance
• withdrawal

Please note: the last two symptoms do not apply to patients taking opioids solely under
appropriate medical supervision.

Definition & Treatment of Chronic Pain


Definition: Pain that persists beyond the expected period of healing. Usually the time
interval is arbitrary and varies depending on the condition.
Approach to treatment:
• Set clear goals with patients. Rarely possible to completely relieve pain, so aim to
achieve a level of pain the patient feels he/she can live with. Measure with
functional goals and numeric assessment of pain level.
• Use non-pharmacologic treatments such as biofeedback for chronic pelvic pain,
physical therapy and cognitive behavioral therapy for chronic back pain.
• When using medications, first select specifically targeted non-opioid
therapies such as anti-epileptic drugs for neuropathic pain or anti-
inflammatories for musculoskeletal pain.
• When using opioids:
- Use long-acting agents along with the other agents and use the lowest possible
dose that improves patients' function.
- Use a pain medication agreement.

Opioids for Treatment of Pain


Most short-acting opioids such as hydrocodone and oxycodone are indicated only for
use in acute pain or for breakthrough pain, when long-acting agents are insufficient to
control symptoms. These drugs should be used cautiously since they cause side effects
like euphoria, hence frequent overuse or diversion for other purposes.
63

For patients with chronic pain, when non-opioid therapies are not sufficient, it is
sometimes appropriate to prescribe low doses of long-acting opioids, along with other
agents to improve function.

Components of a pain medication agreement often include:


• Informed consent, including the likelihood of physical dependence and cognitive
impairment.
• Female patients of childbearing age must understand the implication of becoming
pregnant while on opioids and the likelihood that children will be born physically
dependent.
• Agreement to obtain prescriptions only from the contracting physician and, preferably,
from one designated pharmacy.
• Agreement to take the medication only as prescribed, with provisions built-in that provide
some latitude for the consumption of more or less medication as symptoms dictate.
• Acknowledgment that patients are responsible for their written prescriptions, medication,
and the arranging of refills during regular office hours. They also must plan ahead so as
not to run out of medication during weekends or vacation periods.
• Specified terms for agreement violation, and understanding that lack of adherence may
result in weaning and discontinuation of opioid therapy.

Goals of Headache Treatment


The 2000 US Headache Consortium defined the following goals for preventive
treatment: (1) decrease attack frequency by 50% and decrease intensity and duration;
(2) improve responsiveness to acute therapy; (3) improve function and decrease
disability; and (4) prevent the occurrence of a medication overuse headache (MOH)
and chronic daily headache.

Case 20: 28F with abdominal pain (intimate partner violence)


64
65

Red Flags of Life-Threatening Condition in Patient with Abdominal/Pelvic Pain


There are more than many signs and symptoms of a life-threatening condition in a
patient with abdominal or pelvic pain. Examples include:
• Abrupt onset of severe pain
• Shock with hypotension and tachycardia
• Distension
• Peritoneal irritation signs
• Rigid abdomen
• Pulsatile abdominal mass
• Absent bowel sounds
• Fever
• Vomiting
• Diarrhea
• Weight loss
• Menstrual changes
• Trauma, prior surgeries, or operative scars
• History/presence of blood in emesis
• History/presence of blood in stool
• Severity of the pain
• Ecchymoses/bruising
• Rebound tenderness
• Mass or ascites

Obstetrical History
G Gravida or number of pregnancies
T Number of Term pregnancies
P Number of Preterm infants
A Number of spontaneous or induced Abortions
L Number of Living children

Severe / Life-Threatening Causes of Abdominal Pain


Appendicitis
Patients with appendicitis often start with visceral pain which is dull and in the
periumbilical region; within a short time it classically localizes (presents with) fairly
acute onset of moderate to severe right lower quadrant pain. There is often a history
of nausea and/or vomiting. There are usually some changes in the patient's bowel
movements.

Hepatitis
Although not acutely life-threatening, hepatitis is very important to recognize and
66

diagnose as it can be contagious and some forms of hepatitis can lead to liver cancer.
Patients usually present with nausea, vomiting, diarrhea, light colored stools, and/or
dark urine which is often described as cola- or tea-colored. Patients generally have
fever and yellow discoloration of their eyes, skin and mucus membranes (jaundice).
Patients may have abdominal pain, loss of appetite, and malaise. It is important to
determine the source of the infection. The diagnosis can usually be made by physical
exam. Laboratory tests are helpful in determining the exact diagnosis. Treatment of the
acute illness is generally supportive care. The history may include heavy alcohol
consumption, high-risk behavior such as IV drug use, foreign travel, or multiple
sexual partners.

Ovarian cyst
Patients with an ovarian cyst generally have lower abdominal pain and pelvic pain. The
pain may be extremely severe, especially if there is a ruptured cyst. The pain may be so
severe that the patient will present to the emergency room for evaluation at the time of
rupture. The pain may persist for several weeks, and may be aggravated by intercourse
or strenuous activity.

Pancreatitis
Pancreatitis is generally a moderately severe to severe epigastric pain that often
radiates to the back, and is accompanied by nausea, vomiting and anorexia. There
is usually a history of excessive alcohol use/abuse or a family history of pancreatitis,
although this can also be caused by gallstones, hypertriglyceridemia and other less
common causes. If suspicion is high, laboratory tests (lipase, amylase) and imaging
(abdominal ultrasound or CT scan) are needed to investigate further.

Pelvic inflammatory disease


Patients with pelvic inflammatory disease (PID) might have abdominal or pelvic pain,
which is worse with sexual intercourse or with activities such as running or jumping,
which cause jarring of the pelvic organs. This diagnosis has significant morbidity,
which increases with the severity of the disease and with the length of time to
diagnosis. Studies show that approximately one in four women who had a single
episode of PID later experienced tubal infertility, chronic pelvic pain, or an ectopic
pregnancy, as a result of scarring and adhesions. Tubal adhesions leading to infertility
have been reported to occur in 33% of women after their first episode of PID, and up to
50% after the second pelvic infection.

Normal pregnancy
Women who have normal pregnancies may experience some lower abdominal
discomfort or pain as the uterus undergoes normal growth. This is more a diagnosis of
67

exclusion, but you would not want to miss a pregnancy. Certain medications should not
be given to women who are pregnant. Fetuses should not be exposed to radiation.

Ectopic pregnancy
Ectopic pregnancy is a medical emergency. Early medical treatment reduces the need
for surgery, but if the fallopian tube is in danger of rupture, surgical intervention may be
necessary. Patients present with divergent symptoms ranging from no pain and
normal menses, to intense pain and irregular or absent menses. A good history, the
physical exam and lab testing are crucial for this diagnosis. Imaging is also usually
needed. You need the date of the patient's last menstrual period (LMP), her menstrual
history, most recent intercourse dates, types of contraception used in past few years,
history of any vaginal or pelvic infections, and history of previous ectopic or normal
pregnancies.

Trauma
A careful history is important in regard to trauma. Be aware of the patient's body
language and response to touch. Consider the consistency of the history with the exam.
Have the patient undress and examine the patient thoroughly in a gown so that all
areas can be visualized.

Birth Control Methods


• Oral contraceptives are available in combination type, containing estrogen and progestin,
or progestin only. Both are highly effective against pregnancy when taken regularly
(92-99.7%). These work by inhibiting ovulation and by thickening the cervical
mucus. The most significant risk is thromboembolism and there can be an increased
risk of stroke in patients who are hypertensive, especially if they smoke tobacco.
Benefits include menstrual cycle control, decreased menstrual symptoms and bleeding,
and decreased acne.
• Hormonal implant is a matchstick-sized rod implanted under the skin, which slowly releases
progestin and protects against pregnancy for three years. The mechanism of action
is similar to that of the oral contraceptive. Effectiveness against pregnancy is 99.95%.
Infection at insertion site and depression are the side effects of greatest concern.
• Depo-Provera injection is a progestin injection given once every twelve weeks, which is 97-
99.3% effective against pregnancy. The mechanism of action is similar to the oral
contraceptive. The most common side effect is irregular to absence of vaginal
bleeding, worse in the first six to nine months. Depression, weight gain, and bone
loss are the most concerning complications.
• A hormonal patch is a small beige transdermal patch placed on the skin once a month, and
left in place for three weeks, then removed for the fourth week. Withdrawal bleeding
occurs during the week without the patch. The patch slowly releases estrogen and
progestin. Effectiveness, mechanism of action, and risks are essentially the same as for
oral contraceptives.
68

• NuvaRing is a small ring, 2" in diameter, containing estrogen and progestin. The ring is
inserted vaginally, by the patient, once a month and left in place for three weeks
followed by one week off. Effectiveness, mechanism of action, and risks are the same
as oral contraceptives.
• Intrauterine device (IUD) is a small T-shaped device inserted into the cervix in a medical
office. There are two different types: one contains copper which lasts 10 years
(ParaGard-Copper T), the other contains progestin and is effective for five years
(Mirena). They are 99.2-99.8% effective against pregnancy. Both alter the movement
of the sperm and inhibit the egg from implanting in the uterus. The IUD containing
progestin further suppresses ovulation and thickens the cervical mucus. Increased
menstrual bleeding and dysmenorhea are not uncommon. Uterine perforation and
infection post-insertion are rare, but potential consequences. The initial expense may
be high, especially if IUD is not covered by the patient's insurance. It is preferred that
the patient be in a monogamous relationship.
• Sterilization (male/female) options are permanent surgical procedures. Tubal ligation
(female) is a disruption of the fallopian tubes, preventing the ovum/ova from being
fertilized and transported to the uterus. A vasectomy (male) disrupts the vas deferens,
preventing sperm from reaching the seminal fluid. These are considered to be 98.5-
99.5% effective against pregnancy without user error. Concerns are surgical
complications including infection and failure of female sterilization. Risk of ectopic
pregnancy is increased after a tubal ligation. These procedures can be expensive
initially. Additionally, they are difficult and expensive to reverse if the patient changes
his/her mind. Essure is a form of sterilization in which a coil of metal and polymer is
placed in each proximal fallopian tube through hysteroscope. The coil expands into
place when released. The body mounts an inflammatory response to the device
resulting in fibrosis within weeks of implantation. A follow up hysterosalpingogram
(HSG) is done to confirm infertility. While expensive, the cost is significantly less
expensive than tubal ligation without the risks accompanying anesthesia and surgical
incision. It is reported to be 99.8% effective after confirmation of success by HSG.
• Barrier contraception. The diaphragm and cervical cap are synthetic discs or cups of
varying sizes made out of latex (diaphragm) or silicone (cervical cap) designed to hold
spermicide near the cervical os and prevent sperm from accessing the uterus and
consequently the egg. The device is inserted prior to intercourse and remains in
place until six hours after the last intercourse. They are 84-94% effective against
pregnancy and are highly dependent on user compliance. Potential complications are
vaginal or urinary tract infections and toxic shock syndrome.
• Condom (male/female) is a sheath rolled onto the penile shaft (male type; effectiveness
against pregnancy 85-98%) or inserted into the vagina (female type; effectiveness
against pregnancy 75-95%) just before intercourse. These are safe, protect against
sexually transmissible infections (STIs) and are widely available. The greatest problems
are improper or lack of use. Contraindications are allergy to latex for the male condom,
but other materials are available.
• Spermicides, inserted via applicator, sponge, and suppository, are chemicals which prevent
pregnancy by creating a physical barrier which slows or stops the movement of sperm
69

and prevents sperm from reaching the uterus. These are 68-91% effective against
pregnancy with significant variability secondary to user error, lack of use, and anatomy.
The sponge poses risk for toxic shock syndrome; nonoxyno9 increases risk of
transmission of HIV. These methods are best if used with condoms.
• Withdrawal, also called coitus interuptus, this method requires the withdrawal or manual
removal of the penis from the vagina prior to ejaculation. This method is 73-96%
effective against pregnancy, with variability impacted by the difficulty with timing of
withdrawal and the fact that the pre-ejaculate fluid contains enough sperm to cause
fertilization.
• Fertility awareness encompasses simple and complex methods of predicting fertility based
on ovulation cycle and cervical mucus properties. It is highly dependent on user
compliance and expertise, which helps explain the wide variability in effectiveness.
Sometimes called "the rhythm method;" it is combined with abstinence or with using
another form of birth control on the woman's "unsafe" or fertile days.
• Abstinence is the "simplest" method of birth control. It consists of abstaining from
intercourse. Theoretically, it is 100% effective, but it must be practiced at all times.
• Emergency Contraception are the morning after pill (MAP) and the early contraceptive pill
(ECP) which must be used within 120 hours of unprotected intercourse; the earlier the
better. One of the ways it is felt to work is by making the uterus or endometrium
unfavorable for implantation. Levonorgestrel is available in the United States as Plan B
One-Step, a single dose emergency contraceptive containing levonorgestrel 1.5 mg in
a single tablet and as Plan B pill packs that contain two 0.75 mg tablets to be taken 12
hours apart. Generic formulations for the single tablet regimen are also available (eg,
Next Choice One Dose, My Way). The single 1.5 mg dose regimen is available over the
counter without age restrictions. The two 0.75 mg tablet levonorgestrel regimen is
available only by prescription for those under age 17 years and kept behind the
pharmacy counter to facilitate proof of age prior to purchase. MAP and ECP are not to
be confused with RU 486, more commonly called "the abortion pill."

Preventive Health Measures Recommended for a 28-Year-Old Woman by the U.S.


Preventive Services Task Force
In this visit, which is problem focused, you would not address all these topics, but would rather
keep them in mind as they relate to her problem. Plans should be made for a preventive health
visit in the future.
Health Behaviors for a 28-year-old exam/visit Good Nutrition/ Normal BMI:
• Alcohol Use
• Drug Use
• Tobacco Use
• Exercise
• Safe Sex
• Immunizations
• Screening/Counseling for hyperlipidemia, diabetes
• Screening/Counseling for other risk factors per patient lifestyle
70

• Screening/Counseling for other illnesses per family history

A Recommendations: 
According to USPSTF, there is high certainty that the net benefit is
substantial.
Cervical Women ages 21 to 65 years with cytology (Pap smear) every 3 years or,
Cancer for women ages 30 to 65 years who want to lengthen the screening interval,
Screening screening with a combination of cytology and human papillomavirus (HPV)
testing every 5 years.
Chlamydia Sexually active, nonpregnant women ages 24 and younger OR women
Screening ages 25 and older at increased risk.
HIV Screening Adolescents and adults ages 15 to 65 years. Younger adolescents and older
adults who are at increased risk should also be screened.
All pregnant women, including those who present in labor who are
untested and whose HIV status is unknown.
Syphilis Men and women at increased risk.
Screening

B Recommendations:
According to USPSTF, there is high certainty that the net benefit is moderate or there is
moderate certainty that the net benefit is moderate to substantial.
Alcohol Misuse Adults aged 18 years or older for alcohol misuse and provide persons
Screening and engaged in risky or hazardous drinking with brief behavioral counseling
Behavioral interventions to reduce alcohol misuse.
Counseling:
BRCA Mutation Women at increased risk.
Testing for Breast
and Ovarian Cancer:
Breast Cancer Risk Shared, informed decision-making with women who are at increased
Reduction risk for breast cancer about medications to reduce their risk.
Medications For women who are at increased risk for breast cancer and at low risk
for adverse medication effects, clinicians should offer to prescribe
risk-reducing medications, such as tamoxifen or raloxifene.
Breastfeeding: All pregnant women and new mothers.
Primary Care
Interventions to
Promote
Depression Adults, in clinical practices with systems of care in place.
Screening
Gonorrhea Screening Pregnant women and women with risk factors.
Healthy Diet Adults with hyperlipidemia, elevated blood sugar and other risk
Counseling factors for cerebrovascular disease.
Hepatitis B Virus In persons at high risk for infection.
Infection Screening
71

Hepatitis C Virus In persons at high risk for infection. And one-time screening for
Infection Screening HCV infection to adults born between 1945 and 1965.
Intimate Partner Screen all women of childbearing age for intimate partner violence,
Violence Screening such as domestic violence, and provide or refer women who screen
positive to intervention services.
Lipid Disorders in Women 20-45 if increased risk for coronary heart disease.
Adults Screening
Obesity Screening Screen all adults. Offer or refer patients with a body mass index (BMI)
and Intensive of 30 kg/m2 or higher to intensive, multicomponent behavioral interventions.
Counseling
Sexually Transmitted Sexually active adolescents and adults at increased risk.
Infections Behavioral
Counseling
Type 2 Diabetes Sustained blood pressure 135/80+ mmHg.
Mellitus Screening
Men and Women

Intimate Partner Violence - Screening Recommendations, Prevalence, and


Complications
Screening recommendations:
The American College of Obstetrics and Gynecology suggests screening all patients
who come to them: family planning patients, all ob-gyn patients, and all
prenatal patients, at first visit, at each trimester, and at the post-partum visit. It
may help to preface asking such questions with a statement such as: "Because domestic
violence is so common, I ask all of my patients about this ..."
The USPSTF recommends that clinicians screen women of childbearing age for
intimate partner violence, such as domestic violence, and provide or refer women who
screen positive to intervention services. (Level of Evidence: B)
This is routinely done at annual exams or when red flags are present.
Here is a good resource regarding screening for intimate partner violence.
Prevalence:
It is important to be aware of domestic violence when addressing our patients
as approximately 25% of women in the US report being victimized by an intimate
partner at some point in their lifetime. While the majority of intimate partner
violence (IPV) victims are women, IPV victims are both male and female, occur in both
heterosexual and same sex relationships and cross all socioeconomic, age and ethnic
divides.
Complications:
In addition to the trauma incurred, the rates of chronic disease, including heart disease,
diabetes, depression and suicide is significantly higher in victims as well as in adults
who were victimized as children as a result of direct abuse and exposure to domestic
72

violence.

Red Flags for Intimate Partner Violence


Women who were victimized by their intimate partner are more likely to experience:
• Migraines, frequent headaches
• Chronic pain syndrome
• Heart and blood pressure problems
• Arthritis
• Stomach ulcers, frequent indigestion, diarrhea, constipation, irritable bowel
syndrome, spastic colon
• Pain during sex (dyspareunia), dysmenorrhea, vaginitis, pelvic inflammatory
disease, chronic pelvic pain syndrome, and other gynecological diagnoses
• Invasive cervical cancer and preinvasive cervical neoplasia
• Depression, anxiety and post-traumatic stress
• Unexplained or poorly explained findings on physical exam

Red flags for intimate partner violence include:



• Delay in seeking medical care
• Non-compliance with treatment plan
• Partner insisting on staying close and answering questions directed to patient
• Hesitancy in answering questions or inconsistent or incorrect answers given to
questions
• Shyness or reticence in answering questions
Facilitating Discussion About Domestic Violence
There are several non-judgmental ways to ask about intimate partner violence.
Examples include:
• "Do you feel safe at home?"
• "Because violence is so common, and there are so many forms of violence, I am
asking all my patients about it. Is anyone now or has anyone in the past hurt you
physically or sexually. Is anyone threatening you?"
• "All couples disagree at sometime. What happens when you disagree/fight/argue?"
• "Does your partner ever force you to do things you do not want to do or keep you
from doing things you want to do?"
• "Does your partner control all the money?"
• "I often see the type of symptoms that you have in people who are being hurt at
home or in a relationship. Is this happening to you?"

What to do:
Ask screening questions, create a safe setting, interview the patient alone, ensure
confidentiality, direct assessment (weapons in the house, etc), know local laws,
73

facilitate impartiality, listen non-judgmentally.

Recommended Studies for Evaluation of Abdominal Pain


Pap smear - thin prep
Recommended in the setting of previous abnormal results.
KOH/Saline wet prep
This is a quick test which should be done as it could indicate inflammation (white blood
cells) or diagnose trichomonas, bacterial vaginosis, or yeast vaginitis.
Chlamydia / gonorrhea DNA probe
Chlamydia and gonorrhea can present with a yellow discharge, abdominal pain, and
dyspareunia. This is the preferred method for diagnosis of chlamydia and gonorrhea
because both can be performed with the same sample and the sample can be
endocervical, urethral, vaginal, or urine.
Urine dipstick
A urine dipstick will be helpful to rule out a urinary tract infection (UTI).
Urine pregnancy test
RPR
RPR should be done as part of the STI screen to rule out syphilis.
HIV
HIV should be done as part of the STI screen.

Not indicated:
HPV
Consider ordering a Reflex HPV. Reflex refers to the fact that an abnormal Pap will
automatically be tested for HPV. If the Pap is normal, the HPV testing will not be done.
Pelvic ultrasound
Pelvic exam, urine pregnancy test, and STI testing will guide you in terms of the need
for an ultrasound to evaluate a possible pelvic mass, the size of uterus and ovaries, to
confirm the location of a pregnancy, or to rule out an inflammatory or infectious
process.
Colposcopy
Colposcopy is not indicated until the results of the Pap are back. If the Pap is abnormal,
and/or if HPV is positive, a colposcopy should be scheduled.
Gonorrhea culture
While this is a good test for gonorrhea, a separate test needs to be done on vaginal or
urine samples. However, this is still the preferred method for sexual assault tests, for
tests of cure, and for oral and rectal specimen.
HCG beta sub
This is generally not indicated because of the sensitivity of the urine pregnancy test. If
the results of the urine pregnancy test were inconclusive, a blood test such as HCG Beta
74

Sub would be needed.

Common Symptoms of Exposure to Domestic Violence in Children and Adolescents


• Obvious physical signs of physical or sexual abuse
• Behavioral or emotional problems, such as increased aggression, increased fear or
anxiety, difficulty sleeping or eating, or other signs of emotional distress
• Chronic somatic complaints

30% to 60% of perpetrators of intimate partner violence also abuse children in the
household.

Possible Domestic Violence


You say that while you need to rule out physical causes for Cooper's abdominal pain,
the combination of behavioral and somatic symptoms along with Ms. Bell's
presentation make exposure to domestic violence high on the differential.
Several of Ms. Bell's statements warrant further investigation including:
• Cooper wakes in the middle of the night frequently.
• Ms. Bell has stated that the child is very clingy; confirmed by behavior in exam
room, but within the realm of normal behavior for a toddler.
• Secondary enuresis/bed-wetting.
• Change in behavior regarding nursery school.
• Cooper has missed regular/routine healthcare visits.

You conclude by telling Dr. Nayar that although you feel it is necessary to address these
issues for Cooper, you are not sure the best time to do it. Should you work him in for an
appointment today; have Ms. Bell schedule a later appointment; or address the
concerns today while Ms. Bell is being seen?
Dr. Nayar tells you that you are right to be concerned given the information you have.
He informs you that doing a risk assessment in terms of domestic violence, finding
out about local laws in regard to reporting exposure to domestic violence, and
giving the Ms. Bell access to resources is the key. He reassures you that scheduling a
follow-up for evaluation of Ms. Bell's physical symptoms would be appropriate and
would give an opportunity to re-evaluate Cooper's level of safety.

The Role of the Physician in the Care of a Victim of Domestic Violence


Acknowledge the abuse and health implications: It is important to acknowledge the
abuse, recognize the health implications, and share this with your patient.
Support your patient's decisions: While you may not always agree with the decisions
your patient makes to stay or escape, it is important that you support their decisions.
They have a greater understanding of the complexity of the problem, and have more
75

information on which to base their actions.


Address safety issues: Address the level of risk and safety issues for your patient.
Provide information for them to go to a safe haven if needed. As lack of telephone or
computer (or monitoring of their use) often make it unsafe or impossible for victims to
contact domestic violence resources from home, it is important not only to give
contact information to the patient, but also to offer a means for them to contact
services while in your office.
Practice cultural sensitivity: Cultural differences can give the appearance of abuse, be
accepting of practices some might consider abusive and can inhibit the ability to
interview or support a victim. Practicing sensitivity in caring for patients from different
backgrounds is key to a supportive patient/physician relationship.
Consider impact of abuse on children and other vulnerable parties: When children or
other vulnerable persons who are less able to make decisions on their own behalf are in
the home, consideration must be given to the impact of the abuse on them physically
and mentally as well as their safety. Even in states where reporting abuse towards a
domestic partner is not mandated, the impact or abuse on a child or other vulnerable
person may be and will supersede the desires of the victim to not alert social services.

It is not possible for a clinician to solve the problem of domestic violence for an
individual. Statistically, the most dangerous time for a victim is when they escape
an abusive relationship. While it is hard to accept, sometimes it may be safer for a
victim to stay with the perpetrator. Physicians are not in a position to stop the abuse.
You can make recommendations in terms of decreasing the victim's level of risk by
of providing resources to the patient, limiting access to weapons, and developing
an escape plan with a victim's advocate. Safety planning takes time and expertise.
While some physicians will take the time to be trained to be effective at this, it is
probably best to utilize experts who are associated with domestic violence agencies if
available or to train a staff member to serve this role.

Intimate Partner Violence Safety Assessment


• Increasing severity of violence
• Presence of gun in the house
• Threats to kill or commit suicide by either victim or abuser
• Use of drugs or alcohol by victim or abuser
• Victim trying to leave or left recently
• Harm to children

Increasing Severity of Intimate Partner Violence


• Verbal abuse, insults, yelling
• Throwing things, punching wall
76

• Pushing victim or throwing things at victim


• Slapping
• Kicking, biting
• Hitting with closed fist
• Attempting strangulation
• Beating up; punching with repeated blows
• Threatening with weapon
• Assault with weapon

Ms. Bell is at significant risk because of the history of violence, alcohol use, and
weapons in the house. It isn't clear whether the level of violence is increasing as it is
common for there to be a "honeymoon period" after an episode of violence. Providing
information on resources and options for the victim (E) allows the victim to make the
best choices for themselves.
• A: The clinician's legal responsibility in regard to reporting intimate partner violence
varies from state to state and is highly controversial because of concern that it
might increase the level of risk for the victim. Mandatory reporting is more
consistent in terms of child abuse and contacting social services. Experts in this
field should be sensitive to risks and aware of appropriate responses.

• B: Couple's therapy has been shown to increase the level of risk for the victim. The
victim exposes the abuse, and the perpetrator feels like they are losing control
and tries to regain it through further violence.

• C: Staying with family could be a good option, but would also be an obvious place for
a victim to go and might place the victim and their family at increased risk.

• D: Having someone remove a gun from the house could increase the sense of loss of
control on the part of the perpetrator and accelerate the abusive behavior.

When documenting a history of abusive behavior, use the patient's own words in
quotes and fill in names after pronouns are used. Example: "then he (John
Smith)…". Use neutral language. Example: "patient states", not "patient alleges" which
may give a false impression of disbelief.

Give a detailed description of the patient's appearance, behavioral indicators, injuries


and stages of healing, and health conditions. If the patient consents, use photos to
document injuries; one with a face included in the photo, and then close-ups of
the injury. If photos are not possible, draw and describe injuries on a body map in blue
ink as this is difficult to alter/reproduce. Document recommendations for support and
follow-up as well as materials given to the patient.
Document abuse history as reported by patient in the subjective. The subjective section
is meant to document the patient's experience and verification is not applicable. Include
77

any laboratory and radiology tests ordered with results to maintain a complete record
for the patient. Document results of health and safety assessments and plans for
follow-up as well as referrals and materials given to the patient. Document
recommendations for support. If the patient was referred for a post-rape exam,
document the referral site. Maintain strict confidentiality and safeguard the chart
rather than limit the contents for best care practices.

Case 31: 66F with shortness of breath (Congestive Heart Failure)


Risk Factors for Coronary Artery Disease
Both modifiable and non-modifiable risk factors are important.
Important modifiable risk factors that can be addressed with the patient include:
• diabetes
• high blood pressure, whether controlled or uncontrolled
• current tobacco use
• abnormal lipid levels, particularly high LDL-c and low HDL-c
Key non-modifiable risk factors include:
• older age (men over 45, women over 55)
• family history

Changes in vision (A) and history of dizziness (F) are symptoms that may be secondary
to CAD, but are not risk factors. The history and differential diagnosis for these
symptoms must be considered.
Elevated HDL cholesterol (C) is protective against development of CAD. An HDL-c level
below 40 is considered an independent risk factor.
Myocardial infarction in a 72-year-old first-degree relative (D) does not confer increased
risk. Myocardial infarction (MI) at a young age (male <55; female <65) in a first-
degree relative does increase an individual's risk for CAD.
Female gender (E) is not a risk factor although CAD is the leading cause of death among
women in America. The rise in incidence of cardiovascular disease happens about 10
years later for women than for men.

Prevalence of Coronary Artery Disease


Coronary artery disease (CAD) is the leading cause of death in the United States with
more than a million new and recurrent cardiovascular events occurring each year. With
the aging of the population and the obesity epidemic, its prevalence and impact are
expected to grow.

Acute Coronary Syndrome


The American Heart Association defines acute coronary syndrome as an umbrella term used to
cover any group of clinical symptoms compatible with acute myocardial ischemia. This term
includes unstable angina as well as both ST-segment elevation and non-ST-segment elevation
78

myocardial infarction (STEMI and NSTEMI). Atypical symptoms of ACS are particularly
common in diabetics, women, and the elderly. The most common reason for failure to
diagnose ACS is to ignore noncardiac or atypical symptoms such as dyspnea, fatigue,
nausea, abdominal discomfort, or syncope. If a patient has these symptoms with or without
chest discomfort, it's important to consider that only about 25% of patients who come to the
emergency department with an MI have the 'classic' severe substernal chest pressure.

Interventions to Slow the Progression of Coronary Artery Disease


Glucose control
The desire for tight glucose control (such as hemoglobin A1C of 6% or less in option A)
is understandable. The mortality rate of CAD is higher in patients with diabetes than
in those without diabetes. However, controversy exists regarding appropriate glucose
control for diabetes management. Several guidelines recommend treatment to
reduce HbA1c levels to less than 7%; however, clinical trials have not demonstrated
reductions in cardiovascular events or mortality with intensive glucose control. Studies
have shown inconsistent improvement with intensive glucose control in microvascular
complications, including nephropathy, but increased adverse effects were observed,
including weight gain, fluid retention, and symptomatic hypoglycemia.

Blood pressure control


ACE inhibitors, thiazide-type diuretics, angiotension receptor blockers (ARB), or
calcium channel blockers (CCB) are recommended in JNC-8 for nonblack patients like
Mrs. Hernandez with diabetes.

Note: Only thiazide-type diuretics or CCBs are recommended as first-line agents for
black diabetic patients who require anti-hypertensives. Based on the ACCORD trial of
blood pressure control for diabetics, the JNC-8 recommends all diabetics achieve blood
pressures below 140/90 mmHg (C).

Aspirin
Aspirin has been shown repeatedly to reduce the risk of MI in patients with CAD.
Aspirin is also recommended for primary prevention in some patients (See guidelines
published by the U.S. Preventive Services Task Force (USPSTF). Recently, its benefit in
this setting has been called into question. Here is a nice summary of the benefits and
harms.

Cholesterol control
The ACC/AHA guidelines for cholesterol management recommend that all patients
with type 1 or 2 diabetes age 40-75 years old should be on either a moderate-intensity
statin, or (if their estimated 10-year ASCVD risk >7.5%) a high-intensity statin. To learn
more required information about cholesterol control, visit the MedU Cholesterol
79

Guidelines Module.

Ezetimibe has been demonstrated to be effective in lowering LDL cholesterol.


However, a randomized trial demonstrated that despite the improvements in patients'
cholesterol panels, ezetimibe was ineffective in reducing coronary plaque formation
or coronary events. Thus, it is not recommended for reducing ASCVD risk.

Weight control
Weight loss is generally beneficial for heart disease risk, however rapid changes and
large fluctuations in weight have been associated with an increased risk.

Immunizations
Adult patients with chronic diseases such as CAD should be provided the influenza
vaccine (annually) and the 23-valent pneumococcal vaccine. The CDC now recommends
that such patients receive the pneumovax once prior to the age of 65 and then once
more at age 65. Patients with asplenia or immunocompromise may be given a booster
vaccine five years after their initial pneumovax.

Beta-blockers
Multiple studies have demonstrated the effectiveness of beta-blockers in preventing
second MIs among patients who have had one, and current guidelines recommend
their use whether or not patients have hypertension.

Medications for a Patient with Both Diabetes and Hypertension


Type of Example Recommended use & effects
medication
Angiotensin ramipril Has renal protective effects in diabetics in addition
converting to lowering blood pressure.
enzyme (ACE)
inhibitors
Thiazide hydrochloro Works synergistically with the ACE inhibitor and
diuretics thiazide therefore an effective addition for blood pressure control,
though it can also increase blood sugar.
Oral metformin Metformin is a good choice for an oral hypoglycemic
hypoglycemic given overweight status, although the use of insulin
s mitigates the metformin advantage of not causing weight gain.
NSAID aspirin Aspirin (ASA) recommendations for primary prevention
(ASA) in women are different than those for men. The
recommendation for use of ASA in women is as a preventive
measure for stroke. It is however, not contraindicated in
80

women at risk of cardiac disease. ASA is prescribed for


all patients with CAD and no contraindications.
Statins atorvastatin First line therapy for lowering LDL cholesterol and
reducing ASCVD risk in patients with diabetes.

Physical Exam Findings for Suspected Heart Failure


• Crackles in the lung bases and dullness to percussion
• Jugular venous distention
• Point of maximal impulse (PMI) that is laterally displaced from the mid-clavicular line
towards the axillary line
• S3 from rapid ventricular filling or poor left ventricular functioning
• Enlarged liver, hepato-jugular reflux (distention of the jugular vein upon manual
pressure on the liver), and distention and shifting dullness indicating ascites
• Lower extremity edema and check pulses
• Sacral edema

Differential for New Onset Congestive Heart Failure


Most Likely Diagnoses

Myocardial infarction
A recent myocardial infarction can overwhelm cardiac reserve and result in a new
presentation of CHF.
Acute MI is generally associated with chest pain. However, diabetics may present with
so-called "silent" MI-- painless but evident on EKG.
During an acute MI, patients frequently experience hypotension, complicating the use
of evidenced based acute treatments such as nitrates and beta-blockers, both of which
lower blood pressure.

Arrythmias
Arrhythmias, such as atrial fibrillation, can lead to inadequate filling of the left ventricle
and subsequent heart failure.
Arrhythmias cause heart failure by impeding the forward flow of blood through the
heart.
- Atrial fibrillation and flutter commonly do this particularly when they are associated
with rapid ventricular response. Without the atrial kick the ventricle does not fill as well,
a problem which is exacerbated by the decreased filling time that occurs with
tachycardia.
- Paroxysmal supraventricular tachycardia (PSVT) may also cause this.

Ischemic cardiomyopathy
81

Ischemic cardiomyopathy is the most common cause of CHF and is most often the
result of long-term risk factors such as hypertension, hyperlipidemia, diabetes and
behavioral factors resulting in significant CAD.
Over time, damage to the myocardium and scarring lead to reduced systolic function.

Uncontrolled hypertension
After CAD, the second most common cause of CHD is diastolic dysfunction, often due
to uncontrolled hypertension.

Diagnosing Left Ventricular Hypertrophy


Echocardiography is the test of choice.
There are a variety of criteria for diagnosing LVH by EKG, but their sensitivities are not
excellent. One EKG pattern that is common however is LVH with strain, in which you
see ST depressions (often downward sloping) with T-wave inversions in the lateral
precordial leads. Also, a very large S wave in V3 is strongly suggestive of LVH.

Managing New-Onset Congestive Heart Failure


If your patient has new-onset CHF and you are unsure of the exact precipitating factors
(particularly MI), it is safest to send the patient directly to the emergency room via an
ambulance.
Other options:
• Admitting your patient directly to the floor may lead to an unacceptable delay
in care, as floor nurses have too many patients to manage the initial work-up and
stabilization of an acutely ill patient. Your patient needs an immediate CXR and
urgent stabilization, likely with IV furosemide. If cardiac enzymes are elevated,
the patient will need admission to a coronary care unit (CCU). Otherwise a
regular bed with telemetry cardiac monitoring would be appropriate.
• Managing as an outpatient would be unsafe given her risk of cardiac ischemia.

Radiographic Findings of Congestive Heart Failure


• Cardiomegaly: Defined when the width of the heart is more than half of the width of
the thorax.
• Central vascular congestion and hilar fullness: Patients in failure frequently have
hilar fullness on a PA chest film. Individual vessels may appear enlarged.
• Pleural effusions: Identified by a blunting of the costophrenic angles. This can be
seen posteriorly on a lateral film as well. Occasionally, prominent fluid in the
horizontal fissure will be seen in the right lung.
• Cephalization of pulmonary vasculature: Typically, pulmonary vessels are not well
seen in the upper lung fields. In CHF, however, they become engorged and can
be seen extending from the hilum.
82

Kerley B lines: These are small linear densities 2-3 cm in length seen in the periphery of
the lung fields on the PA view. They represent interstitial fluid in the lung tissue.

Diastolic Dysfunction
Diastolic heart failure occurs when signs and symptoms of heart failure are present, but
left ventricular function is preserved (ejection fraction >45%). It is caused by impaired
LV filling and abnormal LV relaxation and is most commonly related to uncontrolled
hypertension. The incidence of diastolic heart failure increases with age and is more
common in older women. Diastolic heart failure has recently been renamed 'heart
failure with preserved EF' (HFpEF), though the two terms are synonymous.
Concomitant systolic and diastolic dysfunction
As it turns out, all patients with systolic dysfunction also have concomitant diastolic
dysfunction. Therefore, a patient cannot have pure systolic heart failure. On the other
hand, certain cardiovascular diseases such as hypertension may lead to diastolic
dysfunction without concomitant systolic dysfunction. On average, 40% of patients
with CHF have preserved systolic function. These patients have a better prognosis than
those with systolic dysfunction.
Pathophysiology of diastolic dysfunction
The left ventricle develops an abnormality of filling and becomes stiffer and
noncompliant as the disease progresses. Then there is increased pulmonary vessel
pressure during exercise, increased filling pressure and, as left atrial pressure and size
increase, congestion. At this point, exercise intolerance increases and clinical signs of
failure, particularly dyspnea on exertion (DOE), appear. It's like a backup in the pump
with increasing pressure and leakage resulting. So pulmonary congestion, hepatic
congestion and peripheral edema appear.
Differentiating heart failure from non-cardiac conditions in patients with dyspnea
Brain natriuretic peptide (BNP) testing can help differentiate heart failure from non-
cardiac conditions in patients with dyspnea. A normal BNP effectively rules out CHF.
However, an elevated BNP cannot distinguish diastolic from systolic heart failure.

B-Type Natriuretic Peptide (BNP)


B-type natriuretic peptide (BNP) is a substance elaborated in the ventricles in response to
changes in pressure and myocyte stretching that occur when heart failure develops and
worsens. The BNP level in a person with stable heart failure is higher than that in a person with
normal heart function. Natriuretic peptides have been found to have beneficial effects on the
failing heart. These include systemic and pulmonary vasodilation, neurohormonal suppression
(renin-angiotensin system, norepinephrine, aldosterone, endothelin-1), promotion of
natriuresis and diuresis, antiproliferative and antifibrotic effects. In addition BNP does not
increase myocardial oxygen consumption, is not proarrhythmic and does not cause
kaliuresis.
83

Serum BNP Levels (pg/ml) Indicate


< 100 No heart failure
100-300 Suggest heart failure
present
>300 Mild heart failure
>600 Moderate heart failure
>900 Severe heart failure

ACCF/AHA NYHA Functional


Stages of Classification
Heart System
At risk for HF but
A without structural
heart disease or
symptoms of HF
Structural heart I No symptoms and no limitation in ordinary physical activity,
B disease present, e.g. shortness of breath when walking, climbing stairs, etc.
but no signs or Class I may overlap with ACCF/AHA Stage C as well.
symptoms of HF
Structural heart Mild symptoms (mild shortness of breath and/or angina)
C disease with prior II and slight limitation during ordinary activity
or current
symptoms of HF

III Marked limitation in activity due to symptoms, even


during less-than-ordinary activity, e.g. walking short
distances (20-100 meters).
Refractory HF Severe limitations. Experiences symptoms even while
D requiring IV at rest. May regress to Class III,
specialized unlike ACCF/AHA Stage D HF
interventions

Echocardiogram
• Two-dimensional echocardiography measures the size of the chambers, the
thickness of the walls, and the size of the cavity.
• It also evaluates the movement of heart structures.
• Doppler assesses blood flow (direction and velocity) through the valves and
chambers.
• The ratio of left ventricular filling velocities (E/A ratio) is a marker of diastolic
dysfunction. The E/A ratio is reduced when diastolic dysfunction is present.
84

Management of Stage C Heart Failure


The American College of Cardiology Foundation/American Heart Association
Guidelines for the Diagnosis and Management of Heart Failure in Adults were published
in 2005 and updated in 2009 and most recently in 2013. They provide an excellent guide
for the management of systolic heart failure.

ACE inhibitors represent the mainstay of management of systolic heart failure.


Multiple randomized trials have demonstrated reductions in mortality and
hospitalizations among patients treated with these medications. They also have key
roles in the management of both diabetes and coronary artery disease.

ARBs have also been demonstrated to improve mortality in patients with systolic
failure. Given that they are more expensive than ACE inhibitors, they are typically
reserved for patients who can't tolerate ACEs due to side effects such as cough. An ACE
inhibitor should not be combined with an ARB, due to evidence of harms when they
are combined.

Digoxin in randomized trials has demonstrated improved symptoms and reduced


hospitalizations in patients with NYHA class II, III, and IV systolic failure. Classes II and III
fall under the umbrella of Grade C in the newer grading system. Physicians need to be
cautious about digoxin toxicity, particularly in patients with renal insufficiency.

Loop diuretics such as furosemide (Lasix) have a central role in the management of
CHF to improve symptoms in patients with fluid retention. Fluid overload should be
minimized so that other medications (such as ACE inhibitors and beta blockers) can
work better. In patients with diastolic dysfunction (or Heart Failure with Preserved
Ejection Fraction), excessive diuresis can worsen failure by decreasing left
ventricular filling, so diuretics should be used with caution.

Certain Beta-blockers such as metoprolol succinate have a central role in the


management of both diastolic and systolic heart failure. Specifically, bisoprolol,
carvedolol, and sustained-release metoprolol have been shown to reduce mortality in
patients with NYHA Class II and III systolic heart failure (Grade C failure). Physicians
need to have caution when starting beta-blocker therapy, since their negative inotropic
and chronotropic effects can worsen failure initially. Generally, they should not be
started in the setting of decompensated failure, and should be titrated up to maximal
doses very slowly.

Eplerenone was compared to placebo in a 2011 randomized, double-blind trial for


NYHA Class II heart failure. Eplerenone reduced both the risk of death and the risk of
85

hospitalization among patients with systolic heart failure whose ejection fraction was
no more than 35% and who had mild symptoms (EMPHASIS-HF study). Other trials
have demonstrated improvements in mortality for patients with NYHA class III and IV
heart failure who are treated with spironolactone, but the efficacy of this particular
potassium-sparing agent has not been demonstrated in NYHA Class II heart failure.

Calcium channel blockers do not have a major role in the management of heart failure.
Amlodipine (B) has been demonstrated to increase peripheral edema and therefore
may be avoided. In patients with angina, their vasodilatory effects may improve
symptoms.

Assessing Pre-test Probability of Coronary Artery Disease


In general, patients are grouped into three categories of risk:
low risk under 10%
medium everyone
risk else
high risk over 80%

Nuclear Stress Testing


Myocardial perfusion imaging, or nuclear stress testing, shows the extent of perfusion
abnormalities and serves as an independent predictor of cardiac death alone, or cardiac death
and MI, in patients with or without diabetes. The actual test differs from routine exercise stress
testing in that images of the heart are produced and analyzed, giving more detailed
information. The patient either exercises on a treadmill or, if unable to exercise, is given
pharmaceutical stimulation of the heart using persantine, for example. Imaging at rest is done
either before or 1-2 hours after exercise. The thallium or technitium is injected about 1 and 1/2
minutes before the peak of exercise. The patient is then placed in a gamma camera scanner
which produces images which are then analyzed by computer. Areas of abnormal perfusion
or myocardial damage are pictured in the images. The test is useful to help determine: blood
flow to heart muscle at rest and during exercise, extent of damage from an MI, cause of chest
pain (angina), and safe level of exercise for the patient. It also can be used to help determine
clinical management of the patient.

Cardiac Test for Diagnosis of CAD in Patients with Intermediate Pre-Test


Probability
Exercise treadmill testing (ETT)
The ACC/AHA 2002 Guideline Update for Exercise Testing makes the case that all
patients with intermediate risk (i.e. between ~10 and 80% risk) should undergo
exercise treadmill testing (ETT) as the initial diagnostic test for suspected CAD. A
major exception to this rule is for patients in whom the baseline EKG is not
interpretable (which includes those with Wolff-Parkinson-White syndrome, a paced
86

heart rhythm, a left bundle branch block, or more than 1 mm of ST depression at


baseline). Given that Mrs. Hernandez does not have these contraindications, you could
make a case for starting with this test. However, the negative predictive value of a
normal ETT in a patient with reasonably high risk is not very good. For example, in a
patient with a 50% pre-test probability, a negative ETT would lead to a post-test
probability of 30%, which is still too high to effectively rule out CAD. Furthermore,
some have argued that ETTs are less predictive of CAD in women compared to men.

Stress echocardiography and Nuclear stress testing

Stress echocardiography and nuclear stress testing (either using thallium or


sestamibi as the nuclear agent) are both commonly used in patients with intermediate
risk of CAD. Reports vary depending on the patient population and the setting, but
both tests have specificities and particularly sensitivities that are better than those for
ETT. The choice of which of these two tests to order depends heavily on regional
practice patterns and test availability. A 2009 joint statement by multiple cardiology
societies, including the AHA, noted that it was reasonable to jump to a nuclear stress
test (skipping ETT) in a patient with new-onset heart failure.

CAC scoring and CT angiography are newer testing modalities that are not widely used,
each requiring an expensive multidetector-row CT scanner. CT angiography (as well as
MR angiography) is still generally an experimental technology. CAC scoring has been
evaluated for use in the evaluation of patients with chest pain. While its sensitivity is
similar to nuclear stress testing and stress echocardiography, its specificity is very low.
Neither test currently has a role in the diagnosis of CAD.

Management of Diastolic Failure


The management of diastolic failure has been studied less than systolic failure. There
are no randomized trials that demonstrate improvements in mortality for specific
agents in this case.

The general principles of treatment are to minimize fluid overload with diuretics,
control the blood pressure, slow down the heart rate (particularly in patients with
atrial fibrillation), and manage comorbid CHD.

Many physicians start with either a beta-blocker or non-dihydropyridine calcium


channel blocker (such as diltiazem) in order to slow down the heart rate, increase the
ventricular filling time, and decrease the blood pressure.
Excessive diuresis and preload reduction can actually worsen diastolic failure.
87

1. Diastolic heart failure (DHF). Assessment: New onset heart failure with
improvement since admission. Plan: Discuss diagnosis with the patient, review
medications, discuss daily weights, restrict salt in diet, and answer questions.
2. CHD. Assessment: Newly found ischemia that likely contributes to her DHF. Plan:
Continue daily aspirin, beta-blocker, statin. Follow up with cardiology soon to consider
catheterization.
3. Diabetes. Assessment: Previously uncontrolled. Patient now adherent to
medications and determined to continue to follow diet and exercise regimen previously
prescribed. Fingerstick glucose (nonfasting) of 140 mg/dL today indicates better
control. Plan: Continue present regimen, support her in her dietary changes and
adherence to her regimen, follow her HbA1c.
4. Hypertension. Assessment: Controlled with metoprolol, hydrochlorothiazide, and
ramipril. Plan: Continue present regimen.
5. Hyperlipidemia. Assessment: Poorly controlled secondary to recent non-adherence.
Plan: Continue atorvastatin 80 mg, recheck lipids in four weeks to see if adherence
leads to adequate response.
6.Psychosocial stressors. Assessment: Grief reaction, possible depression,
anxiety. Plan: Counsel patient and assess present status. Include a) Loss of spouse with
grief reaction, possible depression b) Family dysfunction with daughter and grandson,
c) Home care needs d) Instruction and support for managing treatment regimen.
7. Preventive care. Assessment: Need for Pap, mammogram, colonoscopy,
immunizations, particularly pneumovax and influenza."

Case 33: 28F with dizziness

Deciding Whether to Use Antibiotic Treatment for Uncomplicated Respiratory Tract


or Ear Infections
Uncomplicated According to American Academy of Pediatrics and American Academy of
otitis media Family Physicians guidelines, children over two years old with
uncomplicated acute otitis media may be treated with additional
observation without prescribing antibiotics; children less than six months
should be treated with antibiotics; and those between six months and two
years with uncomplicated unilateral otitis media may be cautiously
observed first - depending on the certainty of the diagnosis, social supports,
and clinical picture.
Community Community acquired pneumonia should be treated with antibiotics.
acquired
pneumonia
Streptococcal Streptococcal pharyngitis should be treated with antibiotics to prevent
pharyngitis complications and to shorten the course of disease. Penicillin is the
88

treatment of choice.
Maxillary sinusitis Maxillary sinusitis is usually preceded by an upper respiratory infection.
Signs and symptoms include facial pain in the area of the maxillary sinuses,
purulent nasal discharge, post-nasal drip, and tenderness to palpation or
percussion of the sinuses. A recent Cochrane review concluded that in
otherwise uncomplicated maxillary sinusitis, the beneficial effect of
antibiotics is minimal and does not justify the use. This systematic review
excluded studies of sinusitis complicated by involvement of multiple
sinuses, severe systemic signs and symptoms, acute isolated frontal
sinusitis, recurrent sinusitis, or sinusitis with known anatomic defect.
Viral upper Treatment of viral upper respiratory infections with antibiotics does not
respiratory improve prognosis or decrease length of illness.
infections
Bronchitis Whether or not to treat bronchitis with antibiotics is less clear, as antibiotics
can have a modest effect on the length and severity of symptoms in acute
bronchitis. However, most people will recover without antibiotic
treatment. The increasing resistance patterns favor a watchful waiting
approach in the treatment of otherwise healthy individuals with acute
bronchitis.

Characteristic Symptoms of Seasonal Influenza


Fever, cough, myalgias, and sore throat are characteristic symptoms of influenza.

Although diarrhea (A) can be associated with seasonal flu, it is not characteristic, and
when it does occur it is not bloody. Bloody diarrhea would be more likely associated
with a primary gastrointestinal disorder.
Seasonal flu is not contracted through contact with animals (D) despite the name
"swine flu" for the recent H1N1 strain of influenza. Influenza is a respiratory virus spread
through droplet respiratory secretions.
Uncomplicated seasonal flu is not characterized by shortness of breath (E). The
presence of shortness of breath should alert you to the possibility of complications or
alternative diagnoses.

Symptomatic Treatment of Upper Respiratory Infection


• decongestant or saline nasal spray for congestion
• acetaminophen for fever and pain

Dizziness: Presyncope, Disequilibrium, & Vertigo


A precise history is important in order to further differentiate between the potential
etiologies of dizziness.
89

There are three categories to differentiate between when a patient presents with
dizziness: presyncope, disequilibrium, and vertigo.
• Presyncope - Feeling light-headed or faint, as opposed to actually passing out.
Sometimes patients with presyncope feel worse when they stand up quickly.
• Disequilibrium - A feeling of being off balance.
• Vertigo - A sensation of the room spinning

Cardiac arrhythmias and some kinds of valvular heart disease like aortic stenosis can
cause syncope (a temporary loss of consciousness, often described as "fainting" or
"passing out") so those patients may experience dizziness.

Presyncope Symptoms, Etiologies, and Management


Dizziness that is described as "lightheaded" or, "like I'm going to faint," is generally
classified as presyncope and is usually caused by inadequate cerebral
perfusion. Treatment, therefore, is dependent on identifying and correcting the
specific underlying cause.

All of the following conditions cause inadequate perfusion of the central nervous
system via various mechanisms including:
Mechanism Condition Management
inadequate cardiac myocardial infarct
output due to
"pump failure"
inadequate cardiac atrial fibrillation Rate control in atrial fibrillation is achieved through
output due to pharmacologic or electric cardioversion or use of
decreased filling calcium-channel blockers, beta blockers, or digoxin
time if cardioversion is contraindicated or ineffective.
inadequate cardiac tachycardia of Treatment of dizziness due to tachycardia caused by
output due to thyroid storm thyroid storm is focused on treatment of the underlying
decreased filling hyperthyroidism and cardiac rate control with beta blockers.
time
inadequate cardiac bradyarrhythmias Medications are a frequent cause of bradyarrhythmias,
output due to and treatment is simply withdrawal of the medication.
decreased filling Symptomatic bradyarrhythmias frequently require a
time pacemaker.
inadequate cardiac valvular heart For example, aortic stenosis is a common valvular lesion
output due to disease in the elderly that may be asymptomatic; however,
obstruction once syncope develops, valve replacement may be indicated.
• inadequate acute blood loss Replacing volume and raising hemoglobin concentration
cardiac such as gastric are the mainstays of management.
90

output due ulcer bleed


to decreased
preload due
to volume
depletion
• poor cerebral
oxygenation
due to
inadequate
hemoglobin
concentratio
n

Patients with cerebellar degeneration (A) typically describe feeling off balance,
which is a description of disequilibrium. Treatment of cerebellar degeneration is
focused on the underlying pathology. Balance rehabilitation physical therapy may be of
benefit in some patients.

Aminoglycoside toxicity (B) usually causes symptoms of vertigo and hearing loss.
Patients describe their dizziness as if the room is spinning.
Patients with peripheral neuropathy (F) have loss of sensation and, particularly, position
sense. This causes patients to feel disequilibrium, as if they have lost their balance.

Orthostatic Hypotension
• A drop in systolic blood pressure of ≥ 20 mmHg or
• A drop in diastolic blood pressure of ≥ 10 mmHg
• when changing position from supine to standing
• accompanied by feelings of dizziness or light-headedness

Differential of Dizziness
Most Likely Diagnoses
91

Vestibular neuritis • Commonly associated with a recent URI.


• Nystagmus caused by a peripheral lesion such as this
• does not change direction with gaze.
Benign paroxysmal • Causes acute onset vertigo that can be associated with
positional vertigo • nausea and vomiting and intact hearing.
(BPPV) • Vertigo in BPPV is thought to be caused by calcium carbonate
• debris in the semicircular canals.
• BPPV typically causes episodic rather than constant vertigo that is
• triggered by positional change as calcium debris moves within the
• semicircular canals. Symptoms usually resolve several seconds to
• minutes following position change in BPPV.
Vestibular migraine • Vestibular migraine is a variant of migraine that can cause central vertigo.
• Most patients will give a history of previous migraine headaches.
• However, at the time of a vestibular migraine, many patients do not
• have a headache.
Peripheral vs. Central Vertigo
Peripheral Central
Location of problems with the inner ear or central nervous system
pathology vestibular system tends to be more serious conditions than
peripheral lesions
Nystagmus • unidirectional (usually horizontal and • purely horizontal, vertical, or rotational
rotational) and does not change • does not lessen when the patient focuses
direction gaze
• inhibited by fixating on a point and • persists for a longer period
intensifies when fixation is
withdrawn
• Frenzel glasses prevent fixation and
bring out the nystagmus

Less Likely Diagnoses

Anemia Usually presyncopal in nature and described as lightheadedness.


Not associated with nystagmus .
Anxiety Anxiety can occasionally present with dizziness, but the dizzy sensation is usually
vaguely described and also would not be associated with nystagmus.
Cardiac Cardiac arrhythmias are an important cause of dizziness, especially in patients at
arrythmias risk for cardiac disease. Bradyarrhythmias, atrial fibrillation, atrial flutter, heart block,
and tachyarrhythmias are all potential causes of presyncope. Although evidence
suggests that in elderly patients cardiac etiologies of dizziness and even vertigo
are common, cardiac etiologies are not common in younger age groups without
92

any cardiac risk factors and with a normal cardiac exam. In addition, cardiac
etiologies of dizziness would not present with nystagmus.
Cerebellar Cerebellar infarct is an uncommon cause of vertigo and is more often characterized
infarct as disequilibrium. Nystagmus that resolves with gaze fixation is characteristic of
peripheral rather than central causes of vertigo and speaks strongly against this diagnosis.
In addition, patients with a cerebellar infarct will usually demonstrate neurologic findings
localized to the cerebellum such as severe ataxia and/or dysmetria.
Meniere's Episodes of unilateral hearing loss, tinnitus, and vertigo form the classic triad of Meniere's
disease disease.
Orthostatic Orthostatic hypotension can cause dizziness that seems positional since it occurs upon
hypotensio sitting or standing up suddenly. Orthostatic hypotension occurs as a result of volume loss
n in the intravascular space, such as with dehydration or acute blood loss.
Otitis media Otitis media can occasionally be a cause of vertigo. After a recent upper respiratory
infection, otitis media is a possibility; however, patients with otitis media usually
have ear pain and an abnormal ear exam. Fever also commonly accompanies otitis media.
Transient Transient ischemic attack (TIA) can cause symptoms of vertigo but these should
ischemic not be constant.
attack
(TIA)

Common Causes of Vertigo in Primary Practice


• Most common cause: 
- Benign paroxysmal positional vertigo (BPPV)
• Second most common causes:
- Vestibular neuritis results when a viral (or, less
commonly, bacterial) infection of the inner ear causes inflammation of the
vestibular branch of the eighth cranial nerve.

- Acute labyrinthitis occurs
when an infection affects both branches of the nerve resulting in tinnitus
and/or hearing loss as well as vertigo.

Dix-Hallpike Maneuver
One way to confirm the diagnosis of BPPV is the Dix-Hallpike maneuver.
Turn the patient's head to 45 degrees and quickly lay him down supine with his head
just over the end of the exam table. Then turn the head to the side which should
reproduce the symptoms of dizziness and produce nystagmus. Observe for 20 to 30
seconds. If present, the nystagmus will have the fast component in the direction of the
pathology. Next, sit the patient up and observe again for nystagmus.

Head Thrust Test


Observation of nystagmus is essential to differentiating between peripheral and central
vertigo. The head thrust test is used to demonstrate a likely peripheral lesion.
93

Normally, when you face your patient and ask them to keep looking at your nose, his
eyes will stay fixed on your nose if you move his head suddenly to the side. If there is a
peripheral lesion in the vestibular system, the vestibular ocular reflex will be
disrupted and his eyes will move with the head and then saccade back to center
when his head is moved in the direction of the lesion. A normal head thrust test in the
presence of vertigo means the peripheral vestibular system is intact and that the lesion
is central.

When Neuroimaging is Indicated for Patients with Vertigo


Neuroimaging (MRI) is required for patients with vertigo if there is suggestion of a
central lesion.

Imaging
Needed?
A Yes There are multiple reasons to be concerned about a central lesion and possible
infarct in this patient. Her age puts her at risk as does her hypertension.
Her physical exam shows nystagmus that changes direction and that does
not inhibit with focus. Both of these findings are consistent with a central lesion.
She needs an urgent MRI.
B No This patient has a classic history of benign paroxysmal positional vertigo (BPPV).
In addition, the positive Dix-Hallpike maneuver confirms the diagnosis.
Neuroimaging is not required.
C No The triad of recurrent episodes of vertigo, tinnitus, and hearing loss is characteristic
of Meniere's disease which is a peripheral lesion. A positive head thrust test
reassures that the lesion is peripheral.
D No Unidirectional nystagmus that disappears with fixation and recurs with loss
of fixation implies a peripheral lesion. In the absence of other neurological
signs and symptoms in an otherwise well young woman, neuroimaging is not
needed since the likelihood of a central lesion is minimal.
E Yes A normal head thrust test in the face of constant and new vertigo combined
with a history of migraines indicates a possible central lesion. Neuroimaging is needed.

Management of Peripheral Vertigo


Diuretics and a low salt diet are commonly used to treat the vertigo of Menière's
disease following reasoning that use of diuretics will decrease the endolymphatic
pressure and abate symptoms. A 2006 Cochrane review noted that there were "no
trials of high enough quality" to allow a recommendation for or against the use of
diuretics in the treatment of Menière's disease.
The Epley maneuver, or canalith repositioning, is the hallmark of treatment for BPPV.
The etiology of vertigo in BPPV is the presence of calcium debris in the semicircular
94

canals. Through careful positioning of the patient, the Epley maneuver relieves
symptoms by returning the deposits back to the vestibule. The maneuver can be
performed in the clinic and modified by the patient at home. To perform the Epley
maneuver for right-sided symptoms, the patient sits on the exam table with his head
turned 45 degrees to the right. With the clinician supporting the head, the patient
quickly lies back with his head hanging over the exam table supported by the clinician
as in the Dix-Hallpike test. Once the nystagmus has stopped, the clinician turns the
head 90 degrees to the left and the position is held for 30 seconds. Next, the patient
rolls onto his left side, with his face at a 45 degree angle to the floor. This position is
held for 30 more seconds. The patient returns to the sitting position now with his legs
off the left edge of the table. After another 30 seconds, the patient can resume normal
head position. The maneuver can also be repeated on the other side. Repositioning
maneuvers are not effective for the treatment of vertigo not caused by canalith debris.
A Cochrane review found "There is moderate to strong evidence that vestibular
rehabilitation is a safe, effective management for unilateral peripheral vestibular
dysfunction, based on a number of high quality randomized controlled trials." Patients
can be trained in vestibular rehabilitation by a physical therapist.
Vestibular suppressant medications can be effective short-term treatment of vertigo.
Commonly used anticholinergic vestibular suppressants such as meclizine and
dimenhydrinate also have some anti-emetic effects that are useful in controlling the
nausea and vomiting associated with vertigo. Anti-emetics can be a useful adjunct in
select patients. Non-selective phenothiazine anti-emetics, such at metoclopramide and
promethazine, can be effective. Since all these medications can also cause sedation,
they should be used acutely only and avoided in the elderly.

Except in the case of vertigo secondary to otitis media, antimicrobials are not useful in
the treatment of vertigo.

Safety of Vestibular Suppresant Medications While Breastfeeding


The LactMed database states:
• "Occasional doses of meclizine are probably acceptable during breastfeeding. Large
doses or more prolonged use may cause effects in the infant or decrease the milk
supply..."
• "Based on minimal excretion of other phenothiazine derivatives, it appears that
occasional short-term use of promethazine for the treatment of nausea and
vomiting poses little risk to the breastfed infant. With repeated doses, observe
infants for excess sedation."

Вам также может понравиться